OB PRACTICE QUESTIONS

Réussis tes devoirs et examens dès maintenant avec Quizwiz!

Because a pregnant client's diabetes has been poorly controlled throughout her pregnancy, the nurse would be alert for which of the following in the neonate at birth? A) Macrosomia B) Hyperglycemia C) Low birth weight D) Hypobilirubinemia

A) Macrosomia

Which of the following would be most appropriate for the nurse to suggest about pushing to a woman in the second stage of labor? A) "Lying flat with your head elevated on two pillows makes pushing easier." B) "Choose whatever method you feel most comfortable with for pushing." C) " Let me help you decide when it is time to start pushing." D) "Bear down like you're having a bowel movement with every contraction."

B) "Choose whatever method you feel most comfortable with for pushing."

A pregnant woman needs an update in her immunizations. Which of the following vaccinations would the nurse ensure that the woman receives? A) Measles B) Mumps C) Rubella D) Hepatitis B

D) Hepatitis B

The fetus of a nulliparous woman is in a shoulder presentation. The nurse would most likely prepare the client for which type of birth? A) Cesarean B) Vaginal C) Forceps-assisted D) Vacuum extraction

A) Cesarean

A newborn is suspected of having fetal alcohol syndrome. Which of the following would the nurse expect to assess? A) Bradypnea B) Hydrocephaly C) Flattened maxilla D) Hypoactivity

C) Flattened maxilla

The nurse is inspecting the external genitalia of a male newborn. Which of the following would alert the nurse to a possible problem? A) Limited rugae B) Large scrotum C) Palpable testes in scrotal sac D) Absence of engorgement

A) Limited rugae

After teaching the parents of a newborn with retinopathy of prematurity (ROP. about the disorder and treatment, which statement by the parents indicates that the teaching was successful? A) "Can we schedule follow-up eye examinations with the pediatric ophthalmologist now?" B) "We can fix the problem with surgery." C) "We'll make sure to administer eye drops each day for the next few weeks." D) "I'm sure the baby will grow out of it."

A) "Can we schedule follow-up eye examinations with the pediatric ophthalmologist now?"

After teaching a woman who has had an evacuation for a hydatidiform mole (molar pregnancy. about her condition, which of the following statements indicates that the nurse's teaching was successful? A) "I will be sure to avoid getting pregnant for at least 1 year." B) "My intake of iron will have to be closely monitored for 6 months." C) "My blood pressure will continue to be increased for about 6 more months." D) "I won't use my birth control pills for at least a year or two."

A) "I will be sure to avoid getting pregnant for at least 1 year."

After the nurse teaches a client about ways to reduce the symptoms of premenstrual syndrome, which client statement indicates a need for additional teaching? A) "I will make sure to take my estrogen supplements a week before my period." B) "I've signed up for an aerobic exercise class three times a week." C) "I'll cut down on the amount of coffee and colas I drink." D) "I quit smoking about a month ago, so that should help."

A) "I will make sure to take my estrogen supplements a week before my period."

After assessing a woman who has come to the clinic, the nurse suspects that the woman is experiencing dysfunctional uterine bleeding. Which statement by the client would support the nurse's suspicions? A) "I've been having bleeding off and on that's irregular and sometimes heavy." B) "I get sharp pain in my lower abdomen usually starting soon after my period comes." C) "I get really irritable and moody about a week before my period." D) "My periods have been unusually long and heavy lately."

A) "I've been having bleeding off and on that's irregular and sometimes heavy."

During a follow-up prenatal visit, a pregnant woman asks the nurse, "How long do you think I will be in labor?" Which response by the nurse would be most appropriate? A) "It's difficult to predict how your labor will progress, but we'll be there for you the entire time." B) "Since this is your first pregnancy, you can estimate it will be about 10 hours." C) "It will depend on how big the baby is when you go into labor." D) "Time isn't important; your health and the baby's health are key."

A) "It's difficult to predict how your labor will progress, but we'll be there for you the entire time."

A nurse is teaching a client how to perform Kegel exercises. Which of the following would the nurse include? (Select all that apply.) A) "Squeeze your rectal muscles as if you are trying to avoid passing flatus." B) "Tighten your pubococcygeal muscles for a count of 10." C) "Contract and relax your pubococcygeal muscles rapidly 10 times." D) "Try bearing down for about 10 seconds for no more than 5 times." E) "Do these exercises at least 5 times every hour."

A) "Squeeze your rectal muscles as if you are trying to avoid passing flatus."' C) "Contract and relax your pubococcygeal muscles rapidly 10 times."

A woman is scheduled for an anterior and posterior colporrhaphy as treatment for a cystocele. When the nurse is explaining this treatment to the client, which of the following descriptions would be most appropriate to include? A) "This procedure helps to tighten the vaginal wall in the front and back so that your bladder and urethra are in the proper position." B) "Your uterus will be removed through your vagina, helping to relieve the organ that is putting the pressure on your bladder." C) "This is a series of exercises that you will learn to do so that you can strengthen your bladder muscles." D) "These are plastic devices that your physician will insert into your vagina to provide support to the uterus and keep it in the proper position."

A) "This procedure helps to tighten the vaginal wall in the front and back so that your bladder and urethra are in the proper position."

A pregnant woman in her second trimester tells the nurse, "I've been passing a lot of gas and feel bloated." Which of the following suggestions would be helpful for the woman? A) "Watch how much beans and onions you eat." B) "Limit the amount of fluid you drink with meals" C) "Try exercising a little more." D) "Some say that eating mints can help." E) "Cut down on your intake of cheeses."

A) "Watch how much beans and onions you eat." C) "Try exercising a little more." D) "Some say that eating mints can help."

A nurse is describing the different types of regional analgesia and anesthesia for labor to a group of pregnant women. Which statement by the group indicates that the teaching was successful? A) "We can get up and walk around after receiving combined spinal-epidural analgesia." B) "Higher anesthetic doses are needed for patient-controlled epidural analgesia." C) "A pudendal nerve block is highly effective for pain relief in the first stage of labor." D) "Local infiltration using lidocaine is an appropriate method for controlling contraction pain."

A) "We can get up and walk around after receiving combined spinal-epidural analgesia."

A woman is to undergo an amnioinfusion. Which statement would be most appropriate to include when teaching the woman about this procedure? A) "You'll need to stay in bed while you're having this procedure." B) "We'll give you an analgesic to help reduce the pain." C) "After the infusion, you'll be scheduled for a cesarean birth." D) "A suction cup is placed on your baby's head to help bring it out."

A) "You'll need to stay in bed while you're having this procedure."

The nurse is providing care to several pregnant women who may be scheduled for labor induction. The nurse identifies the woman with which Bishop score as having the best chance for a successful induction and vaginal birth? A) 11 B) 8 C) 6 D) 3

A) 11

A biophysical profile has been completed on a pregnant woman. The nurse interprets which score as normal? A) 9 B) 7 C) 5 D) 3

A) 9

A pregnant woman with gestational diabetes comes to the clinic for a fasting blood glucose level. When reviewing the results, the nurse determines that which result indicates good glucose control? A) 90 mg/dL B) 100 mg/dL C) 110 mg /dL D) 120 mg/dL

A) 90 mg/dL

Twenty minutes after birth, a baby begins to move his head from side to side, making eye contact with the mother, and pushes his tongue out several times. The nurse interprets this as indicating which of the following? A) A good time to initiate breast-feeding B) The period of decreased responsiveness preceding sleep C) The need to be alert for gagging and vomiting D) Evidence that the newborn is becoming chilled

A) A good time to initiate breast-feeding

A group of nursing students are reviewing information about cesarean birth. The students demonstrate understanding of the information when they identify which of the following as an appropriate indication? (Select all that apply.. A) Active genital herpes infection B) Placenta previa C) Previous cesarean birth D) Prolonged labor E) Fetal distress

A) Active genital herpes infection B) Placenta previa C) Previous cesarean birth E) Fetal distress

An LGA newborn has a blood glucose level of 30 mg/dL and is exhibiting symptoms of hypoglycemia. Which of the following would the nurse do next? A) Administer intravenous glucose immediately. B) Feed the newborn 2 ounces of formula. C) Initiate blow-by oxygen therapy. D) Place the newborn under a radiant warmer.

A) Administer intravenous glucose immediately.

A woman comes to the clinic because she has been unable to conceive. When reviewing the woman's history, which of the following would the nurse least likely identify as a possible risk factor? A) Age of 25 years B) History of smoking C) Diabetes since age 15 years D) Weight below standard for height and age

A) Age of 25 years

A client with trichomoniasis is to receive metronidazole (Flagyl). The nurse instructs the client to avoid which of the following while taking this drug? A) Alcohol B) Nicotine C) Chocolate D) Caffeine

A) Alcohol

When assessing a woman in her first trimester, which emotional response would the nurse most likely expect to find? A) Ambivalence B) Introversion C) Acceptance D) Emotional lability

A) Ambivalence

A group of students are reviewing the signs of pregnancy. The students demonstrate understanding of the information when they identify which as presumptive signs? (Select all that apply.) A) Amenorrhea B) Nausea C) Abdominal enlargement D) Braxton-Hicks contractions E) Fetal heart sounds

A) Amenorrhea B) Nausea

Which of the following is a priority when caring for a woman during the fourth stage of labor? A) Assessing the uterine fundus B) Offering fluids as indicated C) Encouraging the woman to void D) Assisting with perineal care

A) Assessing the uterine fundus

The nurse strokes the lateral sole of the newborn's foot from the heel to the ball of the foot when evaluating which reflex? A) Babinski B) Tonic neck C) Stepping D) Plantar grasp

A) Babinski

The nurse determines that a woman has implemented prescribed therapy for her fibrocystic breast disease when the client reports that she has eliminated what from her diet? A) Caffeine B) Cigarettes C) Dairy products D) Sweets

A) Caffeine

Which of the following would the nurse have readily available for a client who is receiving magnesium sulfate to treat severe preeclampsia? A) Calcium gluconate B) Potassium chloride C) Ferrous sulfate D) Calcium carbonate

A) Calcium gluconate

When preparing the discharge teaching plan for the woman who had surgery to correct pelvic organ prolapse, which of the following would the nurse include? A) Care of the indwelling catheter at home B) Emphasis on coughing to prevent complications C) Return to usual activity level in a few days D) Daily douching with dilute vinegar solution

A) Care of the indwelling catheter at home

After teaching a group of nursing students about variations in newborn head size and appearance, the instructor determines that the teaching was successful when the students identify which of the following as a normal variation? (Select all that apply.) A) Cephalhematoma B) Molding C) Closed fontanels D) Caput succedaneum E) Posterior fontanel diameter 1.5 cm

A) Cephalhematoma B) Molding D) Caput succedaneum

A client's membranes spontaneously ruptured, as evidenced by a gush of clear fluid with a contraction. Which of the following would the nurse do next? A) Check the fetal heart rate. B) Perform a vaginal exam. C) Notify the physician immediately. D) Change the linen saver pad.

A) Check the fetal heart rate.

The nurse interprets which of the following as evidence that a client is in the taking-in phase? A) Client states, "He has my eyes and nose." B) Client shows interest in caring for the newborn. C) Client performs self-care independently. D) Client confidently cares for the newborn.

A) Client states, "He has my eyes and nose."

A nurse is developing a plan of care for a preterm newborn to address the nursing diagnosis of risk for delayed development. Which of the following would the nurse include? (Select all that apply.) A) Clustering care to promote rest B) Positioning newborn in extension C) Using kangaroo care D) Loosely covering the newborn with blankets E) Providing nonnutritive sucking

A) Clustering care to promote rest C) Using kangaroo care E) Providing nonnutritive sucking

After teaching a group of students about the different methods for contraception, the instructor determines that the teaching was successful when the students identify which of the following as a mechanical barrier method? (Select all that apply.) A) Condom B) Cervical cap C) Cervical sponge D) Diaphragm E) Vaginal ring

A) Condom B) Cervical cap C) Cervical sponge D) Diaphragm

A 10-week pregnant woman with diabetes has a glycosylated hemoglobin (HbA1C. level of 13%. At this time the nurse should be most concerned about which of the following possible fetal outcomes? A) Congenital anomalies B) Incompetent cervix C) Placenta previa D) Abruptio placentae

A) Congenital anomalies

The nurse notes persistent early decelerations on the fetal monitoring strip. Which of the following would the nurse do next? A) Continue to monitor the FHR because this pattern is benign. B) Perform a vaginal exam to assess cervical dilation and effacement. C) Stay with the client while reporting the finding to the physician. D) Administer oxygen after turning the client on her left side.

A) Continue to monitor the FHR because this pattern is benign.

A nurse is discussing fetal development with a pregnant woman. The woman is 12 weeks pregnant and asks, "What's happening with my baby?" Which of the following would the nurse integrate into the response? (Select all that apply.) A) Continued sexual differentiation B) Eyebrows forming C) Startle reflex present D) Digestive system becoming active E) Lanugo present on the head

A) Continued sexual differentiation D) Digestive system becoming active

A nurse is explaining to the parents of a child with bladder exstrophy about the care their infant requires. Which of the following would the nurse include in the explanation? (Select all that apply.) A) Covering the area with a sterile, clear, nonadherent dressing B) Irrigating the surface with sterile saline twice a day C) Monitoring drainage through the suprapubic catheter D) Administering prescribed antibiotic therapy E) Preparing for surgical intervention in about 2 weeks

A) Covering the area with a sterile, clear, nonadherent dressing C) Monitoring drainage through the suprapubic catheter D) Administering prescribed antibiotic therapy

A client states that she is to have a test to measure bone mass to help diagnose osteoporosis. The nurse would most likely plan to prepare the client for: A) DEXA scan B) Ultrasound C) MRI D) Pelvic x-ray

A) DEXA scan

A nurse suspects that a pregnant client may be experiencing abruption placenta based on assessment of which of the following? (Select all that apply.) A) Dark red vaginal bleeding B) Insidious onset C) Absence of pain D) Rigid uterus E) Absent fetal heart tones

A) Dark red vaginal bleeding D) Rigid uterus E) Absent fetal heart tones

A nursing instructor is preparing a class on newborn adaptations. When describing the change from fetal to newborn circulation, which of the following would the instructor most likely include? (Select all that apply.) A) Decrease in right atrial pressure leads to closure of the foramen ovale. B) Increase in oxygen levels leads to a decrease in systemic vascular resistance. C) Onset of respirations leads to a decrease in pulmonary vascular resistance. D) Increase in pressure in the left atrium results from increases in pulmonary blood flow. E) Closure of the ductus venosus eventually forces closure of the ductus arteriosus.

A) Decrease in right atrial pressure leads to closure of the foramen ovale. C) Onset of respirations leads to a decrease in pulmonary vascular resistance. D) Increase in pressure in the left atrium results from increases in pulmonary blood flow. E) Closure of the ductus venosus eventually forces closure of the ductus arteriosus.

A nurse suspects that a postpartum client is experiencing postpartum psychosis. Which of the following would most likely lead the nurse to suspect this condition? A) Delirium B) Feelings of anxiety C) Sadness D) Insomnia

A) Delirium

A woman who is 42 weeks pregnant comes to the clinic. Which of the following would be most important? A) Determining an accurate gestational age B) Asking her about the occurrence of contractions C) Checking for spontaneous rupture of membranes D) Measuring the height of the fundus

A) Determining an accurate gestational age

Assessment of a postpartum woman experiencing postpartum hemorrhage reveals mild shock. Which of the following would the nurse expect to assess? (Select all that apply.) A) Diaphoresis B) Tachycardia C) Oliguria D) Cool extremities E) Confusion

A) Diaphoresis, D) Cool extremities

A woman who is 12 hours postpartum had a pulse rate around 80 beats per minute during pregnancy. Now, the nurse finds a pulse of 60 beats per minute. Which of these actions should the nurse take? A) Document the finding, as it is a normal finding at this time. B) Contact the physician, as it indicates early DIC. C) Contact the physician, as it is a first sign of postpartum eclampsia. D) Obtain an order for a CBC, as it suggests postpartum anemia.

A) Document the finding, as it is a normal finding at this time.

While changing a female newborn's diaper, the nurse observes a mucus-like, slightly bloody vaginal discharge. Which of the following would the nurse do next? A) Document this as pseudomenstruation B) Notify the practitioner immediately C) Obtain a culture of the discharge D) Inspect for engorgement

A) Document this as pseudomenstruation

A pregnant woman undergoes maternal serum alpha-fetoprotein (MSAFP) testing at 16 to 18 weeks' gestation. Which of the following would the nurse suspect if the woman's level is decreased? A) Down syndrome B) Sickle-cell anemia C) Cardiac defects D) Open neural tube defect

A) Down syndrome

After teaching a pregnant woman with iron deficiency anemia about nutrition, the nurse determines that the teaching was successful when the woman identifies which of the following as being good sources of iron in her diet? (Select all that apply.) A) Dried fruits B) Peanut butter C) Meats D) Milk E) White bread

A) Dried fruits B) Peanut butter C) Meats

After determining that a newborn is in need of resuscitation, which of the following would the nurse do first? A) Dry the newborn thoroughly B) Suction the airway C) Administer ventilations D) Give volume expanders

A) Dry the newborn thoroughly

After the birth of a newborn, which of the following would the nurse do first to assist in thermoregulation? A) Dry the newborn thoroughly. B) Put a hat on the newborn's head. C) Check the newborn's temperature. D) Wrap the newborn in a blanket.

A) Dry the newborn thoroughly.

A multipara client develops thrombophlebitis after delivery. Which of the following would alert the nurse to the need for immediate intervention? A) Dyspnea, diaphoresis, hypotension, and chest pain B) Dyspnea, bradycardia, hypertension, and confusion C) Weakness, anorexia, change in level of consciousness, and coma D) Pallor, tachycardia, seizures, and jaundice

A) Dyspnea, diaphoresis, hypotension, and chest pain

A postpartum client is experiencing subinvolution. When reviewing the woman's labor and birth history, which of the following would the nurse identify as being least significant to this condition? A) Early ambulation B) Prolonged labor C) Large fetus D) Use of anesthetics

A) Early ambulation

When developing the plan of care for the parents of a newborn, the nurse identifies interventions to promote bonding and attachment based on the rationale that bonding and attachment are most supported by which measure? A) Early parent-infant contact following birth B) Expert medical care for the labor and birth C) Good nutrition and prenatal care during pregnancy D) Grandparent involvement in infant care after birth

A) Early parent-infant contact following birth

A woman has just entered the second stage of labor. The nurse would focus care on which of the following? A) Encouraging the woman to push when she has a strong desire to do so B) Alleviating perineal discomfort with the application of ice packs C) Palpating the woman's fundus for position and firmness D) Completing the identification process of the newborn with the mother

A) Encouraging the woman to push when she has a strong desire to do so

A postpartum woman is diagnosed with metritis. The nurse interprets this as an infection involving which of the following? (Select all that apply.) A) Endometrium B) Decidua C) Myometrium D) Broad ligament E) Ovaries F) Fallopian tubes

A) Endometrium, B) Decidua, C) Myometrium

Assessment of a pregnant woman reveals a pigmented line down the middle of her abdomen. The nurse documents this as which of the following? A) Linea nigra B) Striae gravidarum C) Melasma D) Vascular spiders

A) Linea nigra

A woman who delivered a healthy newborn several hours ago asks the nurse, "Why am I perspiring so much?" The nurse integrates knowledge that a decrease in which hormone plays a role in this occurrence? A) Estrogen B) hCG C) hPL D) Progesterone

A) Estrogen

A nurse is preparing a presentation for a group of women at the clinic who have been diagnosed with genital herpes. Which of the following would the nurse expect to include as a possible precipitating factor for a recurrent outbreak? (Select all that apply.) A) Exposure to ultraviolet light B) Exercise C) Use of corticosteroids D) Emotional stress E) Sexual intercourse.

A) Exposure to ultraviolet light C) Use of corticosteroids D) Emotional stress E) Sexual intercourse.

A newborn with severe meconium aspiration syndrome (MAS. is not responding to conventional treatment. Which of the following would the nurse anticipate as possibly necessary for this newborn? A) Extracorporeal membrane oxygenation (ECMO) B) Respiratory support with a ventilator C) Insertion of a laryngoscope for deep suctioning D) Replacement of an endotracheal tube via x-ray

A) Extracorporeal membrane oxygenation (ECMO)

The nurse is performing Leopold's maneuvers to determine fetal presentation, position, and lie. Which action would the nurse do first? A) Feel for the fetal buttocks or head while palpating the abdomen. B) Feel for the fetal back and limbs as the hands move laterally on the abdomen. C) Palpate for the presenting part in the area just above the symphysis pubis. D) Determine flexion by pressing downward toward the symphysis pubis.

A) Feel for the fetal buttocks or head while palpating the abdomen.

A nurse palpates a woman's fundus to determine contraction intensity. Which of the following would be most appropriate for the nurse to use for palpation? A) Finger pads B) Palm of the hand C) Finger tips D) Back of the hand

A) Finger pads

Assessment of a fetus identifies the buttocks as the presenting part, with the legs extended upward. The nurse identifies this as which type of breech presentation? A) Frank B) Full C) Complete D) Footling

A) Frank

A nurse is preparing a teaching program for a group of pregnant women about preventing infections during pregnancy. When describing measures for preventing cytomegalovirus infection, which of the following would the nurse most likely include? A) Frequent handwashing B) Immunization C) Prenatal screening D) Antibody titer screening

A) Frequent handwashing

A woman comes to the clinic and tells the nurse that she has read an article about certain foods that have anticancer properties and help boost the immune system. The nurse identifies A) Garlic B) Soybeans C) Milk D) Leeks E) Flax seeds

A) Garlic B) Soybeans D) Leeks E) Flax seeds

A nurse is describing advances in genetics to a group of students. Which of the following would the nurse least likely include? A) Genetic diagnosis is now available as early as the second trimester. B) Genetic testing can identify presymptomatic conditions in children. C) Gene therapy can be used to repair missing genes with normal ones. D) Genetic agents may be used in the future to replace drugs.

A) Genetic diagnosis is now available as early as the second trimester.

A pregnant woman tests positive for HBV. Which of the following would the nurse expect to administer? A) HBV immune globulin B) HBV vaccine C) Acylcovir D) Valacyclovir

A) HBV immune globulin

A nurse is developing a teaching plan for the parents of a newborn. When describing the neurologic development of a newborn to his parents, the nurse would explain that the development occurs in which fashion? A) Head-to-toe B) Lateral-to-medial C) Outward-to-inward D) Distal-to-caudal

A) Head-to-toe

As part of discharge planning, the nurse refers a woman to Reach to Recovery. This group's primary purpose is to: A) Help support women who have undergone mastectomies B) Raise funds to support early breast cancer detection programs C) Provide all supplies needed after breast surgery for no cost D) Collect statistics for research for the American Cancer Society

A) Help support women who have undergone mastectomies

Assessment of a postpartum client reveals a firm uterus with bright-red bleeding and a localized bluish bulging area just under the skin at the perineum. The woman also is complaining of significant pelvic pain and is experiencing problems with voiding. The nurse suspects which of the following? A) Hematoma B) Laceration C) Bladder distention D) Uterine atony

A) Hematoma

In a woman who is suspected of having a ruptured ectopic pregnancy, the nurse would expect to assess for which of the following as a priority? A) Hemorrhage B) Jaundice C) Edema D) Infection

A) Hemorrhage

While assessing a pregnant woman, the nurse suspects that the client may be at risk for hydramnios based on which of the following? (Select all that apply.) A) History of diabetes B) Complaints of shortness of breath C) Identifiable fetal parts on abdominal palpation D) Difficulty obtaining fetal heart rate E) Fundal height below that for expected gestataional age

A) History of diabetes B) Complaints of shortness of breath D) Difficulty obtaining fetal heart rate

A nurse is reviewing the medical record of a postpartum client. The nurse identifies that the woman is at risk for a postpartum infection based on which of the following? (Select all that apply.) A) History of diabetes B) Labor of 12 hours C) Rupture of membranes for 16 hours D) Hemoglobin level 10 mg/dL E) Placenta requiring manual extraction

A) History of diabetes, D) Hemoglobin level 10 mg/dL, E) Placenta requiring manual extraction

Assessment of a newborn reveals a heart rate of 180 beats/minute. To determine whether this finding is a common variation rather than a sign of distress, what else does the nurse need to know? A) How many hours old is this newborn? B) How long ago did this newborn eat? C) What was the newborn's birth weight? D) Is acrocyanosis present?

A) How many hours old is this newborn?

When describing genetic disorders to a group of childbearing couples, the nurse would identify which as an example of an autosomal dominant inheritance disorder? A) Huntington's disease B) Sickle cell disease C) Phenylketonuria D) Cystic fibrosis

A) Huntington's disease

A nurse is assessing a pregnant woman on a routine checkup. When assessing the woman's gastrointestinal tract, which of the following would the nurse expect to find? (Select all that apply.) A) Hyperemic gums B) Increased peristalsis C) Complaints of bloating D) Heartburn E) Nausea

A) Hyperemic gums C) Complaints of bloating D) Heartburn E) Nausea

When assessing a newborn 1 hour after birth, the nurse measures an axillary temperature of 95.8° F, an apical pulse of 114 beats/minute, and a respiratory rate of 60 breaths/minute. Which nursing diagnosis takes highest priority? A) Hypothermia related to heat loss during birthing process B) Impaired parenting related to addition of new family member C) Risk for deficient fluid volume related to insensible fluid loss D) Risk for infection related to transition to extrauterine environment

A) Hypothermia related to heat loss during birthing process

13. A nurse is counseling a mother about the immunologic properties of breast milk. The nurse integrates knowledge of immunoglobulins, emphasizing that breast milk is a major source of which immunoglobulin? A) IgA B) IgG C) IgM D) IgE

A) IgA

The nurse is assessing a preterm newborn who is in the neonatal intensive care unit (NICU. for signs and symptoms of overstimulation. Which of the following would the nurse be least likely to assess? A) Increased respirations B) Flaying hands C) Periods of apnea D) Decreased heart rate

A) Increased respirations

The nurse is reviewing the monitoring strip of a woman in labor who is experiencing a contraction. The nurse notes the time the contraction takes from its onset to reach its highest intensity. The nurse interprets this time as which of the following? A) Increment B) Acme C) Peak D) Decrement

A) Increment

A client experienced prolonged labor with prolonged premature rupture of membranes. The nurse would be alert for which of the following in the mother and the newborn? A) Infection B) Hemorrhage C) Trauma D) Hypovolemia

A) Infection

A new mother asks the nurse, "Why has my baby lost weight since he was born?" The nurse integrates knowledge of which of the following when responding to the new mother? A) Insufficient calorie intake B) Shift of water from extracellular space to intracellular space C) Increase in stool passage D) Overproduction of bilirubin

A) Insufficient calorie intake

A nurse is preparing a class for pregnant women about labor and birth. When describing the typical movements that the fetus goes through as it travels through the passageway, which of the following would the nurse most likely include? (Select all that apply.) A) Internal rotation B) Abduction C) Descent D) Pronation E) Flexion

A) Internal rotation C) Descent E) Flexion

A woman diagnosed with breast cancer is to receive trastuzumab. Which of the following would the nurse incorporate into the explanation about how this drug works? A) It blocks the effect of the HER-2/neu protein inhibiting the growth of cancer cells. B) The drug blocks the conversion of androgens to estrogens C) It interferes with hormone receptors that allow estrogen to enter a cell D) The drug ultimately attacks areas where micrometastasis has occurred.

A) It blocks the effect of the HER-2/neu protein inhibiting the growth of cancer cells.

A woman in the 34th week of pregnancy says to the nurse, "I still feel like having intercourse with my husband." The woman's pregnancy has been uneventful. The nurse responds based on the understanding that: A) It is safe to have intercourse at this time. B) Intercourse at this time is likely to cause rupture of membranes. C) There are other ways that the couple can satisfy their needs. D) Intercourse at this time is likely to result in premature labor.

A) It is safe to have intercourse at this time.

Assessment of a woman in labor reveals cervical dilation of 3 cm, cervical effacement of 30%, and contractions occurring every 7 to 8 minutes, lasting about 40 seconds. The nurse determines that this client is in: A) Latent phase of the first stage B) Active phase of the first stage C) Transition phase of the first stage D) Perineal phase of the second stage

A) Latent phase of the first stage

When teaching a class of pregnant women about the effects of substance abuse during pregnancy, which of the following would the nurse most likely include? A) Low-birth-weight infants B) Excessive weight gain C) Higher pain tolerance D) Longer gestational periods

A) Low-birth-weight infants

A home health care nurse is assessing a postpartum woman who was discharged 2 days ago. The woman tells the nurse that she has a low-grade fever and feels "lousy." Which of the following findings would lead the nurse to suspect metritis? (Select all that apply.) A) Lower abdominal tenderness B) Urgency C) Flank pain D) Breast tenderness E) Anorexia

A) Lower abdominal tenderness, E) Anorexia

During a clinical breast examination, the nurse palpates a well-defined, firm, mobile lump in a 60-year-old woman's left breast. The nurse notifies the physician. Which of the following would the nurse anticipate the physician to order next? A) Mammogram B) Hormone receptor status C) Fine-needle aspiration D) Genetic testing for BRCA

A) Mammogram

After reviewing a client's history, which factor would the nurse identify as placing her at risk for gestational hypertension? A) Mother had gestational hypertension during pregnancy. B) Client has a twin sister. C) Sister-in-law had gestational hypertension. D) This is the client's second pregnancy.

A) Mother had gestational hypertension during pregnancy.

After teaching a group of nursing students about tocolytic therapy, the instructor determines that the teaching was successful when they identify which drug as being used for tocolysis? (Select all that apply.) A) Nifedipine B) Terbutaline C) Dinoprostone D) Misoprostol E) Indomethacin

A) Nifedipine B) Terbutaline E) Indomethacin

A client expresses concern that her 2-hour-old newborn is sleepy and difficult to awaken. The nurse explains that this behavior indicates which of the following? A) Normal progression of behavior B) Probable hypoglycemia C) Physiological abnormality D) Inadequate oxygenation

A) Normal progression of behavior

A woman just delivered a healthy term newborn. Upon assessing the umbilical cord, the nurse would identify which of the following as normal? (Select all that apply.) A) One vein B) Two veins C) One artery D) Two arteries E) One ligament F) Two ligaments

A) One vein

A nurse has been invited to speak at a local high school about adolescent pregnancy. When developing the presentation, the nurse would incorporate information related to which of the following? (Select all that apply.) A) Peer pressure to become sexually active B) Rise in teen birth rates over the years. C) Latinas as having the highest teen birth rate D) Loss of self-esteem as a major impact E) Majority of teen pregnancies in the 15-17-year-old age group

A) Peer pressure to become sexually active C) Latinas as having the highest teen birth rate D) Loss of self-esteem as a major impact

After teaching a group of students about the factors affecting the labor process, the instructor determines that the teaching was successful when the group identifies which of the following as a component of the true pelvis? (Select all that apply.) A) Pelvic inlet B) Cervix C) Mid pelvis D) Pelvic outlet E) Vagina F) Pelvic floor muscles

A) Pelvic inlet C) Mid pelvis D) Pelvic outlet

To assist the woman in regaining control of the urinary sphincter for urinary incontinence, the nurse should teach the client to do which of the following? A) Perform Kegel exercises daily. B) Void every hour while awake. C) Limit her intake of fluid. D) Take a laxative every night.

A) Perform Kegel exercises daily.

Which of the following would the nurse include when teaching women about preventing pelvic support disorders? A) Performing Kegel isometric exercises B) Consuming low-fiber diets C) Using hormone replacement D) Voiding every 2 hours

A) Performing Kegel isometric exercises

The nurse is assisting a postpartum woman out of bed to the bathroom for a sitz bath. Which of the following would be a priority? A) Placing the call light within her reach B) Teaching her how the sitz bath works C) Telling her to use the sitz bath for 30 minutes D) Cleaning the perineum with the peri-bottle

A) Placing the call light within her reach

A nurse is developing a plan of care for a newborn with omphalocele. Which of the following would the nurse include? A) Placing the newborn into a sterile drawstring bowel bag B) Using clean technique for dressing changes C) Preparing the newborn for incision and drainage D) Instituting gavage feedings

A) Placing the newborn into a sterile drawstring bowel bag

A woman comes to the prenatal clinic suspecting that she is pregnant, and assessment reveals probable signs of pregnancy. Which of the following would be included as part of this assessment? (Select all that apply.) A) Positive pregnancy test B) Ultrasound visualization of the fetus C) Auscultation of a fetal heart beat D) Ballottement E) Absence of menstruation F) Softening of the cervix

A) Positive pregnancy test D) Ballottement F) Softening of the cervix

After undergoing diagnostic testing, a woman is diagnosed with a corpus luteum cyst. The nurse anticipates that the woman will require: A) Biopsy B) No treatment C) Oral contraceptives D) Glucophage

B) No treatment

A nurse is preparing a presentation for a group of pregnant women about the labor experience. Which of the following would the nurse most likely include when discussing measures to promote coping for a positive labor experience? (Select all that apply.) A) Presence of a support partner B) View of birth as a stressor C) Low anxiety level D) Fear of loss of control E) Participation in a pregnancy exercise program

A) Presence of a support partner C) Low anxiety level E) Participation in a pregnancy exercise program

In a client's seventh month of pregnancy, she reports feeling "dizzy, like I'm going to pass out, when I lie down flat on my back." The nurse integrates which of the following in to the explanation? A) Pressure of the gravid uterus on the vena cava B) A 50% increase in blood volume C) Physiologic anemia due to hemoglobin decrease D) Pressure of the presenting fetal part on the diaphragm

A) Pressure of the gravid uterus on the vena cava

When planning the care for an SGA newborn, which action would the nurse determine as a priority? A) Preventing hypoglycemia with early feedings B) Observing for respiratory distress syndrome C) Promoting bonding between the parents and the newborn D) Monitoring vital signs every 2 hours

A) Preventing hypoglycemia with early feedings

A nurse is preparing a class for a group of women at a family planning clinic about contraceptives. When describing the health benefits of oral contraceptives, which of the following would the nurse most likely include? (Select all that apply.) A) Protection against pelvic inflammatory disease B) Reduced risk for endometrial cancer C) Decreased risk for depression D) Reduced risk for migraine headaches E) Improvement in acne

A) Protection against pelvic inflammatory disease B) Reduced risk for endometrial cancer E) Improvement in acne

A group of students are reviewing risk factors associated with postpartum hemorrhage. The students demonstrate understanding of the information when they identify which of the following as associated with uterine tone? (Select all that apply.) A) Rapid labor B) Retained blood clots C) Hydramnios D) Operative birth E) Fetal malpostion

A) Rapid labor, C) Hydramnios

A nurse is reading a journal article about care of the woman with pelvic organ prolapse. The nurse would expect to find information related to which of the following? (Select all that apply.) A) Rectocele B) Fecal incontinence C) Cystocele D) Urinary incontinence E) Enterocele

A) Rectocele' C) Cystocele E) Enterocele

After teaching a group of students about the use of antiretroviral agents in pregnant women who are HIV-positive, the instructor determines that the teaching was successful when the group identifies which of the following as the underlying rationale? A) Reduction in viral loads in the blood B) Treatment of opportunistic infections C) Adjunct therapy to radiation and chemotherapy D) Can cure acute HIV/AIDS infections

A) Reduction in viral loads in the blood

A woman in labor received an opioid close to the time of birth. The nurse would assess the newborn for which of the following? A) Respiratory depression B) Urinary retention C) Abdominal distention D) Hyperreflexia

A) Respiratory depression

A newborn is experiencing cold stress. Which of the following would the nurse expect to assess? (Select all that apply.) A) Respiratory distress B) Decreased oxygen needs C) Hypoglycemia D) Metabolic alkalosis E) Jaundice

A) Respiratory distress C) Hypoglycemia E) Jaundice

While reviewing a newborn's medical record, the nurse notes that the chest x-ray shows a ground glass pattern. The nurse interprets this as indicative of: A) Respiratory distress syndrome B) Transient tachypnea of the newborn C) Asphyxia D) Persistent pulmonary hypertension

A) Respiratory distress syndrome

Prior to discharging a 24-hour-old newborn, the nurse assesses her respiratory status. Which of the following would the nurse expect to assess? A) Respiratory rate 45, irregular B) Costal breathing pattern C) Nasal flaring, rate 65 D) Crackles on auscultation

A) Respiratory rate 45, irregular

A client with polycystic ovarian syndrome (PCOS. is receiving oral contraceptives as part of her treatment plan. The nurse understands that the rationale for this therapy is to: A) Restore menstrual regularity B) Induce ovulation C) Improve insulin uptake D) Alleviate hirsutism

A) Restore menstrual regularity

While caring for a preterm newborn receiving oxygen therapy, the nurse monitors the oxygen therapy duration closely based on the understanding that the newborn is at risk for which of the following? A) Retinopathy of prematurity B) Metabolic acidosis C) Infection D) Cold stress

A) Retinopathy of prematurity

A pregnant woman is scheduled to undergo percutaneous umbilical blood sampling. When discussing this test with the woman, the nurse reviews what can be evaluated with the specimens collected. Which of the following would the nurse include? (Select all that apply.) A) Rh incompatibility B) Fetal acid-base status C) Sex-linked disorders D) Enzyme deficiencies E) Coagulation studies

A) Rh incompatibility B) Fetal acid-base status E) Coagulation studies

After teaching a group of adolescents about HIV, the nurse asks them to identify the major means by which adolescents are exposed to the virus. The nurse determines that the teaching was successful when the group identifies which of the following? A) Sexual intercourse B) Sharing needles for IV drug use C) Perinatal transmission D) Blood transfusion

A) Sexual intercourse

A nurse suspects that a preterm newborn is having problems with thermal regulation. Which of the following would support the nurse's suspicion? (Select all that apply.) A) Shallow, slow respirations B) Cyanotic hands and feet C) Irritability D) Hypertonicity E) Feeble cry

A) Shallow, slow respirations B) Cyanotic hands and feet E) Feeble cry

Which action would be most appropriate for the nurse to take when a newborn has an unexpected anomaly at birth? A) Show the newborn to the parents as soon as possible while explaining the defect. B) Remove the newborn from the birthing area immediately. C) Inform the parents that there is nothing wrong at the moment. D) Tell the parents that the newborn must go to the nursery immediately.

A) Show the newborn to the parents as soon as possible while explaining the defect.

The nurse notifies the obstetrical team immediately because the nurse suspects that the pregnant woman may be exhibiting signs and symptoms of amniotic fluid embolism. Which findings would the nurse most likely assess? (Select all that apply.) A) Significant difficulty breathing B) Hypertension C) Tachycardia D) Pulmonary edema E) Bleeding with bruising

A) Significant difficulty breathing C) Tachycardia D) Pulmonary edema E) Bleeding with bruising

After teaching a group of students about fetal heart rate patterns, the instructor determines the need for additional teaching when the students identify which of the following as indicating normal fetal acid-base status? (Select all that apply.) A) Sinusoidal pattern B) Recurrent variable decelerations C) Fetal bradycardia D) Absence of late decelerations E) Moderate baseline variability

A) Sinusoidal pattern B) Recurrent variable decelerations C) Fetal bradycardia

A woman with polycystic ovary syndrome tells the nurse, "I hate this disease. Just look at me! I have no hair on the front of my head but I've got hair on my chin and upper lip. I don't feel like a woman anymore." Further assessment reveals breast atrophy and increased muscle mass. Which nursing diagnosis would most likely be a priority? A) Situational low self-esteem related to masculinization effects of the disease B) Social isolation related to feelings about appearance C) Risk for suicide related to effects of condition and fluctuating hormone levels D) Ineffective peripheral tissue perfusion related to effects of disease on vasculature

A) Situational low self-esteem related to masculinization effects of the disease

A woman using the cervical mucus ovulation method of fertility awareness reports that her cervical mucus looks like egg whites. The nurse interprets this as which of the following? A) Spinnbarkeit mucus B) Purulent mucus C) Postovulatory mucus D) Normal preovulation mucus

A) Spinnbarkeit mucus

A nursing instructor is describing common problems associated with preterm birth. When describing the preterm newborn's risk for perinatal asphyxia, the instructor includes which of the following as contributing to the newborn's risk? (Select all that apply.) A) Surfactant deficiency B) Placental deprivation C) Immaturity of the respiratory control centers D) Decreased amounts of brown fat E) Depleted glycogen stores

A) Surfactant deficiency C) Immaturity of the respiratory control centers

A nurse is describing how the fetus moves through the birth canal. Which of the following would the nurse identify as being most important in allowing the fetal head to move through the pelvis? A) Sutures B) Fontanelles C) Frontal bones D) Biparietal diameter

A) Sutures

A client is to receive an implantable contraceptive. The nurse describes this contraceptive as containing: A) Synthetic progestin B) Combined estrogen and progestin C) Concentrated spermicide D) Concentrated estrogen

A) Synthetic progestin

The nurse is developing a discharge teaching plan for a postpartum woman who has developed a postpartum infection. Which of the following would the nurse most likely include in this teaching plan? (Select all that apply.) A) Taking the prescribed antibiotic until it is finished B) Checking temperature once a week C) Washing hands before and after perineal care D) Handling perineal pads by the edges E) Directing peribottle to flow from back to front

A) Taking the prescribed antibiotic until it is finished, C) Washing hands before and after perineal care, D) Handling perineal pads by the edges

After teaching a group of nursing students about the different types of chemotherapeutic agents used to treat breast cancer, the instructor determines that the teaching was successful when the students identify which of the following as an example of a selective estrogen receptor modulator (SERM)? (Select all that apply.) A) Tamoxifen B) Letozole C) Raloxifene D) Exemestane E) Anastrozole

A) Tamoxifen C) Raloxifene

A woman who is HIV-positive is receiving HAART and is having difficulty with compliance. To promote adherence, which of the following areas would be most important to assess initially? A) The woman's beliefs and education B) The woman's financial situation and insurance C) The woman's activity level and nutrition D) The woman's family and living arrangements

A) The woman's beliefs and education

After teaching a group of students about ovarian cysts, the instructor determines that the teaching was successful when the students identify which type of cyst as being associated with hydatiform mole? A) Theca-lutein cyst B) Corpus luteum cyst C) Follicular cyst D) Polycystic ovarian syndrome

A) Theca-lutein cyst

When assessing the postpartum woman, the nurse uses indicators other than pulse rate and blood pressure for postpartum hemorrhage based on the knowledge that: A) These measurements may not change until after the blood loss is large B) The body's compensatory mechanisms activate and prevent any changes C) They relate more to change in condition than to the amount of blood lost D) Maternal anxiety adversely affects these vital signs

A) These measurements may not change until after the blood loss is large

The nurse is assessing a newborn and suspects that the newborn was exposed to drugs in utero because the newborn is exhibiting signs of neonatal abstinence syndrome. Which of the following would the nurse expect to assess? (Select all that apply.) A) Tremors B) Diminished sucking C) Regurgitation D) Shrill, high-pitched cry E) Hypothermia F) Frequent sneezing

A) Tremors C) Regurgitation D) Shrill, high-pitched cry F) Frequent sneezing

After teaching a group of students about the discomforts of pregnancy, the students demonstrate understanding of the information when they identify which as common during the first trimester? (Select all that apply.) A) Urinary frequency B) Breast tenderness C) Cravings D) Backache E) Leg cramps

A) Urinary frequency B) Breast tenderness C) Cravings

A woman comes to the clinic complaining of a vaginal discharge. The nurse suspects trichomoniasis based on which of the following? (Select all that apply.) A) Urinary frequency B) Yellow/green discharge C) Joint pain D) Blister-like lesions E) Muscle aches

A) Urinary frequency B) Yellow/green discharge

A postpartum woman who is breast-feeding tells the nurse that she is experiencing nipple pain. Which of the following would be least appropriate for the nurse to suggest? A) Use of a mild analgesic about 1 hour before breast-feeding B) Application of expressed breast milk to the nipples C) Application of glycerin-based gel to the nipples D) Reinstruction about proper latching-on technique

A) Use of a mild analgesic about 1 hour before breast-feeding

A pregnant woman is receiving misoprostol to ripen her cervix and induce labor. The nurse assesses the woman closely for which of the following? A) Uterine hyperstimulation B) Headache C) Blurred vision D) Hypotension

A) Uterine hyperstimulation

Which of the following would indicate to the nurse that the placenta is separating? A) Uterus becomes globular B) Fetal head is at vaginal opening C) Umbilical cord shortens D) Mucous plug is expelled

A) Uterus becomes globular

Which position would be most appropriate for the nurse to suggest as a comfort measure to a woman who is in the first stage of labor? (Select all that apply.) A) Walking with partner support B) Straddling with forward leaning over a chair C) Closed knee-chest position D) Rocking back and forth with foot on chair E) Supine with legs raised at a 90-degree angle

A) Walking with partner support B) Straddling with forward leaning over a chair D) Rocking back and forth with foot on chair

A nurse is assessing a newborn who has been classified as small for gestational age. Which of the following would the nurse expect to find? (Select all that apply.) A) Wasted extremity appearance B) Increased amount of breast tissue C) Sunken abdomen D) Adequate muscle tone over buttocks E) Narrow skull sutures

A) Wasted extremity appearance C) Sunken abdomen E) Narrow skull sutures

A newborn was diagnosed with a congenital heart defect and will undergo surgery at a later time. The nurse is teaching the parents about signs and symptoms that need to be reported. The nurse determines that the parents have understood the instructions when they state that they will report which of the following? (Select all that apply.) A) Weight loss B) Pale skin C) Fever D) Absence of edema E) Increased respiratory rate

A) Weight loss C) Fever E) Increased respiratory rate

After teaching a group of students about fetal development, the instructor determines that the teaching was successful when the students identify which of the following as providing the barrier to other sperm after fertilization? A) Zona pellucida B) Zygote C) Cleavage D) Morula

A) Zona pellucida

A woman gives birth to a newborn at 36 weeks' gestation. She tells the nurse, "I'm so glad that my baby isn't premature." Which response by the nurse would be most appropriate? A) "You are lucky to have given birth to a term newborn." B) "We still need to monitor him closely for problems." C) "How do you feel about delivering your baby at 36 weeks?" D) "Your baby is premature and needs monitoring in the NICU."

B) "We still need to monitor him closely for problems."

Which of the following would not be considered a risk factor for bronchopulmonary dysplasia (chronic lung disease)? A) Preterm birth (less than 32 weeks) B) Female gender C) White race D) Sepsis

B) Female gender

The nurse is developing a presentation for a community group of young adults discussing fetal development and pregnancy. The nurse would identify that the sex of offspring is determined at the time of: A) Meiosis B) Fertilization C) Formation of morula D) Oogenesis

B) Fertilization

A woman telephones her health care provider and reports that her "water just broke." Which suggestion by the nurse would be most appropriate? A) "Call us back when you start having contractions." B) "Come to the clinic or emergency department for an evaluation." C) "Drink 3 to 4 glasses of water and lie down." D) "Come in as soon as you feel the urge to push."

B) "Come to the clinic or emergency department for an evaluation."

A 42-year-old woman is scheduled for a mammogram. Which of the following would the nurse include when teaching the woman about the procedure? A) "The room will be darkened throughout the procedure." B) "Each breast will be firmly compressed between two plates." C) "Make sure to refrain from eating or drinking after midnight." D) "A small needle will be inserted to get a sample for evaluation."

B) "Each breast will be firmly compressed between two plates."

A postpartum woman who has experienced diastasis recti asks the nurse about what to expect related to this condition. Which response by the nurse would be most appropriate? A) "You'll notice that this will fade to silvery lines." B) "Exercise will help to improve the muscles." C) "Expect the color to lighten somewhat." D) "You'll notice that your shoe size will increase."

B) "Exercise will help to improve the muscles."

Which statement would alert the nurse to the potential for impaired bonding between mother and newborn? A) "You have your daddy's eyes." B) "He looks like a frog to me." C) "Where did you get all that hair?" D) "He seems to sleep a lot."

B) "He looks like a frog to me."

A postpartum woman who developed deep vein thrombosis is being discharged on anticoagulant therapy. After teaching the woman about this treatment, the nurse determines that additional teaching is needed when the woman states which of the following? A) "I will use a soft toothbrush to brush my teeth." B) "I can take ibuprofen if I have any pain." C) "I need to avoid drinking any alcohol." D) "I will call my health care provider if my stools are black and tarry."

B) "I can take ibuprofen if I have any pain."

A woman with placenta previa is being treated with expectant management. The woman and fetus are stable. The nurse is assessing the woman for possible discharge home. Which statement by the woman would suggest to the nurse that home care might be inappropriate? A) "My mother lives next door and can drive me here if necessary." B) "I have a toddler and preschooler at home who need my attention." C) "I know to call my health care provider right away if I start to bleed again." D) "I realize the importance of following the instructions for my care."

B) "I have a toddler and preschooler at home who need my attention."

A nurse is teaching a women with genital ulcers how to care for them. Which statement by the client indicates a need for additional teaching? A) "I need to wash my hands after touching any of the ulcers." B) "I need to abstain from intercourse primarily when the lesions are present." C) "I should avoid applying ice or heat to my genital area." D) "I can try lukewarm sitz baths to help ease the discomfort."

B) "I need to abstain from intercourse primarily when the lesions are present."

After teaching a woman with pelvic organ prolapse about dietary and lifestyle measures, which of the following statements would indicate the need for additional teaching? A) "If I wear a girdle, it will help support the muscles in the area." B) "I should take up jogging to make sure I exercise enough." C) "I will try to drink at least 64 oz of fluid each day." D) "I need to increase the amount of fiber I eat every day."

B) "I should take up jogging to make sure I exercise enough."

A neonate born to a mother who was abusing heroin is exhibiting signs and symptoms of withdrawal. Which of the following would the nurse assess? (Select all that apply.) A) Low whimpering cry B) Hypertonicity C) Lethargy D) Excessive sneezing E) Overly vigorous sucking F) Tremors

B) Hypertonicity D) Excessive sneezing F) Tremors

After teaching a woman how to perform breast self-examination, which statement would indicate that the nurse's instructions were successful? A) "I should lie down with my arms at my side when looking at my breasts." B) "I should use the fingerpads of my three middle fingers to apply pressure to my breast." C) "I don't need to check under my arm on that side if my breast feels fine." D) "I need to work from the center of my breast outward toward my shoulder."

B) "I should use the fingerpads of my three middle fingers to apply pressure to my breast."

After teaching a postpartum woman about breast-feeding, the nurse determines that the teaching was successful when the woman states which of the following? A) "I should notice a decrease in abdominal cramping during breast-feeding." B) "I should wash my hands before starting to breast-feed." C) "The baby can be awake or sleepy when I start to feed him." D) "The baby's mouth will open up once I put him to my breast."

B) "I should wash my hands before starting to breast-feed."

A nurse is assessing a postpartum client who is at home. Which statement by the client would lead the nurse to suspect that the client may be developing postpartum depression? A) "I just feel so overwhelmed and tired." B) "I'm feeling so guilty and worthless lately." C) "It's strange, one minute I'm happy, the next I'm sad." D) "I keep hearing voices telling me to take my baby to the river."

B) "I'm feeling so guilty and worthless lately."

After teaching a couple about what to expect with their planned cesarean birth, which statement indicates the need for additional teaching? A) "Holding a pillow against my incision will help me when I cough." B) "I'm going to have to wait a few days before I can start breast-feeding." C) "I guess the nurses will be getting me up and out of bed rather quickly." D) "I'll probably have a tube in my bladder for about 24 hours or so."

B) "I'm going to have to wait a few days before I can start breast-feeding."

A postpartum client who is bottle feeding her newborn asks, "When should my period return?" Which response by the nurse would be most appropriate? A) "It's difficult to say, but it will probably return in about 2 to 3 weeks." B) "It varies, but you can estimate it returning in about 7 to 9 weeks." C) "You won't have to worry about it returning for at least 3 months." D) "You don't have to worry about that now. It'll be quite a while."

B) "It varies, but you can estimate it returning in about 7 to 9 weeks."

After teaching the parents of a newborn with periventricular hemorrhage about the disorder and treatment, which statement by the parents indicates that the teaching was successful? A) "We'll make sure to cover both of his eyes to protect them." B) "Our newborn could develop a learning disability later on." C) "Once the bleeding ceases, there won't be any more worries." D) "We need to get family members to donate blood for transfusion."

B) "Our newborn could develop a learning disability later on."

After reviewing information about postpartum blues, a group of students demonstrate understanding when they state which of the following about this condition? A) "Postpartum blues is a long-term emotional disturbance." B) "Sleep usually helps to resolve the blues." C) "The mother loses contact with reality." D) "Extended psychotherapy is needed for treatment."

B) "Sleep usually helps to resolve the blues."

A nurse is teaching postpartum client and her partner about caring for their newborn's umbilical cord site. Which statement by the parents indicates a need for additional teaching? A) "We can put him in the tub to bathe him once the cord falls off and is healed." B) "The cord stump should change from brown to yellow." C) "Exposing the stump to the air helps it to dry." D) "We need to call the doctor if we notice a funny odor."

B) "The cord stump should change from brown to yellow."

A nursing instructor is describing the advantages and disadvantages associated with newborn circumcision to a group of nursing students. Which statement by the students indicates effective teaching? A) "Sexually transmitted infections are more common in circumcised males." B) "The rate of penile cancer is less for circumcised males." C) "Urinary tract infections are more easily treated in circumcised males." D) "Circumcision is a risk factor for acquiring HIV infection."

B) "The rate of penile cancer is less for circumcised males."

A new mother is changing the diaper of her 20-hour-old newborn and asks why the stool is almost black. Which response by the nurse would be most appropriate? A) "You probably took iron during your pregnancy." B) "This is meconium stool, normal for a newborn." C) "I'll take a sample and check it for possible bleeding." D) "This is unusual and I need to report this."

B) "This is meconium stool, normal for a newborn."

After describing continuous internal electronic fetal monitoring to a laboring woman and her partner, which of the following would indicate the need for additional teaching? A) "This type of monitoring is the most accurate method for our baby." B) "Unfortunately, I'm going to have to stay quite still in bed while it is in place." C) "This type of monitoring can only be used after my membranes rupture." D) "You'll be inserting a special electrode into my baby's scalp."

B) "Unfortunately, I'm going to have to stay quite still in bed while it is in place."

A nurse is teaching new parents about bathing their newborn. The nurse determines that the teaching was successful when the parents state which of the following? A) "We can put a tiny bit of lotion on his skin and then rub it in gently." B) "We should avoid using any kind of baby powder." C) "We need to bathe him at least four to five times a week." D) "We should clean his eyes after washing his face and hair."

B) "We should avoid using any kind of baby powder."

After teaching a woman with a postpartum infection about care after discharge, which client statement indicates the need for additional teaching? A) "I need to call my doctor if my temperature goes above 100.4° F." B) "When I put on a new pad, I'll start at the back and go forward." C) "If I have chills or my discharge has a strange odor, I'll call my doctor." D) "I'll point the spray of the peribottle so the water flows front to back."

B) "When I put on a new pad, I'll start at the back and go forward."

A woman is scheduled to undergo fetal nuchal translucency testing. Which of the following would the nurse include when describing this test? A) "A needle will be inserted directly into the fetus's umbilical vessel." B) "You'll have an intravaginal ultrasound to measure fluid in the fetus." C) "The doctor will take a sample of fluid from your bag of waters." D) "A small piece of tissue from the fetal part of the placenta is taken."

B) "You'll have an intravaginal ultrasound to measure fluid in the fetus."

Which of the following would the nurse include when teaching a pregnant woman about chorionic villus sampling? A) "The results should be available in about a week." B) "You'll have an ultrasound first and then the test." C) "Afterwards, you can resume your exercise program." D) "This test is very helpful for identifying spinal defects."

B) "You'll have an ultrasound first and then the test."

A woman who gave birth 24 hours ago tells the nurse, "I've been urinating so much over the past several hours." Which response by the nurse would be most appropriate? A) "You must have an infection, so let me get a urine specimen." B) "Your body is undergoing many changes that cause your bladder to fill quickly." C) "Your uterus is not contracting as quickly as it should." D) "The anesthesia that you received is wearing off and your bladder is working again."

B) "Your body is undergoing many changes that cause your bladder to fill quickly."

A nurse is assisting in the resuscitation of a newborn. The nurse would expect to stop resuscitation efforts when the newborn has no heartbeat and respiratory effort after which time frame? A) 5 minutes B) 10 minutes C) 15 minutes D) 20 minutes

B) 10 minutes

A postpartum woman who is bottle-feeding her newborn asks the nurse, "About how much should my newborn drink at each feeding?" The nurse responds by saying that to feel satisfied, the newborn needs which amount at each feeding? A) 1 to 2 ounces B) 2 to 4 ounces C) 4 to 6 ounces D) 6 to 8 ounces

B) 2 to 4 ounces

Assessment of a newborn's head circumference reveals that it is 34 cm. The nurse would suspect that this newborn's chest circumference would be: A) 30 cm B) 32 cm C) 34 cm D) 36 cm

B) 32 cm

A woman is receiving magnesium sulfate as part of her treatment for severe preeclampsia. The nurse is monitoring the woman's serum magnesium levels. Which level would the nurse identify as therapeutic? A) 3.3 mEq/L B) 6.1 mEq/L C) 8.4 mEq/L D) 10.8 mEq/L

B) 6.1 mEq/L

When describing programs for breast cancer screening, the nurse include breast self-examination (BSE). Which of the following most accurately reflects the current thinking about breast self-examination? A) BSE is essential for early breast cancer detection. B) A woman performing BSE has breast awareness. C) BSE plays a minimal role in detecting breast cancer D) A clinical breast exam has replaced BSE.

B) A woman performing BSE has breast awareness.

A pregnant woman is admitted with premature rupture of the membranes. The nurse is assessing the woman closely for possible infection. Which of the following would lead the nurse to suspect that the woman is developing an infection? (Select all that apply.) A) Fetal bradycardia B) Abdominal tenderness C) Elevated maternal pulse rate D) Decreased C-reactive protein levels E) Cloudy malodorous fluid

B) Abdominal tenderness C) Elevated maternal pulse rate E) Cloudy malodorous fluid

The nurse is assessing a newborn's eyes. Which of the following would the nurse identify as normal? (Select all that apply.) A) Slow blink response B) Able to track object to midline C) Transient deviation of the eyes D) Involuntary repetitive eye movement E) Absent red reflex

B) Able to track object to midline C) Transient deviation of the eyes D) Involuntary repetitive eye movement

Which of the following would lead the nurse to suspect that a postpartum woman is experiencing a problem? A) Elevated white blood cell count B) Acute decrease in hematocrit C) Increased levels of clotting factors D) Pulse rate of 60 beats/minute

B) Acute decrease in hematocrit

The nurse would be least likely to find which of the following in a client with uterine fibroids? A) Regularly shaped, shrunken uterus B) Acute pelvic pain C) Menorrhagia D) Complaints of bloating

B) Acute pelvic pain

A nurse is assessing a client for possible risk factors for chlamydia and gonorrhea. Which of the following would the nurse identify? A) Asian American ethnicity B) Age under 25 years C) Married D) Consistent use of barrier contraception

B) Age under 25 years

After teaching a group of students about fetal development, the instructor determines that the teaching was successful when the students identify which of the following as essential for fetal lung development? A) Umbilical cord B) Amniotic fluid C) Placenta D) Trophoblasts

B) Amniotic fluid

The parents of a newborn become concerned when they notice that their baby seems to stop breathing for a few seconds. After confirming the parents' findings by observing the newborn, which of the following actions would be most appropriate? A) Notify the health care provider immediately. B) Assess the newborn for signs of respiratory distress. C) Reassure the parents that this is an expected pattern. D) Tell the parents not to worry since his color is fine.

B) Assess the newborn for signs of respiratory distress.

When developing the plan of care for a newborn with an acquired condition, which of the following would the nurse include to promote participation by the parents? A) Use verbal instructions primarily for explanations B) Assist with decision making process C) Provide personal views about their decisions D) Encourage them to refrain from showing emotions

B) Assist with decision making process

A pregnant woman admitted to the labor and birth suite undergoes rapid HIV testing and is found to be HIV-positive. Which of the following would the nurse expect to include when developing a plan of care for this women? (Select all that apply.) A) Administration of penicillin G at the onset of labor B) Avoidance of scalp electrodes for fetal monitoring C) Refraining from obtaining fetal scalp blood for pH testing D) Adminstering zidovudine at the onset of labor. E) Electing for the use of forceps-assisted delivery

B) Avoidance of scalp electrodes for fetal monitoring C) Refraining from obtaining fetal scalp blood for pH testing D) Adminstering zidovudine at the onset of labor.

A newborn has an Apgar score of 6 at 5 minutes. Which of the following is the priority? A) Initiating IV fluid therapy B) Beginning resuscitative measures C) Promoting kangaroo care D) Obtaining a blood culture

B) Beginning resuscitative measures

When preparing a schedule of follow-up visits for a pregnant woman with chronic hypertension, which of the following would be most appropriate? A) Monthly visits until 32 weeks, then bi-monthly visits B) Bi-monthly visits until 28 weeks, then weekly visits C) Monthly visits until 20 weeks, then bi-monthly visits D) Bi-monthly visits until 36 weeks, then weekly visits

B) Bi-monthly visits until 28 weeks, then weekly visits

The nurse is assessing the newborn of a mother who had gestational diabetes. Which of the following would the nurse expect to find? (Select all that apply.) A) Pale skin color B) Buffalo hump C) Distended upper abdomen D) Excessive subcutaneous fat E) Long slender neck

B) Buffalo hump C) Distended upper abdomen D) Excessive subcutaneous fat

When describing the stages of labor to a pregnant woman, which of the following would the nurse identify as the major change occurring during the first stage? A) Regular contractions B) Cervical dilation C) Fetal movement through the birth canal D) Placental separation

B) Cervical dilation

A woman in labor is to receive continuous internal electronic fetal monitoring. The nurse reviews the woman's medical record to ensure which of the following as being required? A) Intact membranes B) Cervical dilation of 2 cm or more C) Floating presenting fetal part D) A neonatologist to insert the electrode

B) Cervical dilation of 2 cm or more

A group of students are reviewing information about STIs. The students demonstrate understanding of the information when they identify which of the following as the most common bacterial STI in the United States? A) Gonorrhea B) Chlamydia C) Syphilis D) Candidiasis

B) Chlamydia

The nurse places a warmed blanket on the scale when weighing a newborn. The nurse does so to minimize heat loss via which mechanism? A) Evaporation B) Conduction C) Convection D) Radiation

B) Conduction

After teaching a class about hepatic system adaptations after birth, the instructor determines that the teaching was successful when the class identifies which of the following as the process of changing bilirubin from a fat-soluble product to a water-soluble product? A) Hemolysis B) Conjugation C) Jaundice D) Hyperbilirubinemia

B) Conjugation

A group of nursing students are preparing a presentation for their class about measures to prevent toxoplasmosis. Which of the following would the students be least likely to include? Select all that apply. A) Washing raw fruits and vegetables before eating them B) Cooking all meat to an internal temperature of 140° F C) Wearing gardening gloves when working in the soil D) Avoiding contact with a cat's litter box.

B) Cooking all meat to an internal temperature of 140° F

Which of the following would the nurse include in the plan of care for a newborn receiving phototherapy? A) Keeping the newborn in the supine position B) Covering the newborn's eyes while under the bililights C) Ensuring that the newborn is covered or clothed D) Reducing the amount of fluid intake to 8 ounces daily

B) Covering the newborn's eyes while under the bililights

A nurse is developing a program for pregnant women with diabetes about reducing complications. Which factor would the nurse identify as being most important in helping to reduce the maternal/fetal/neonatal complications associated with pregnancy and diabetes? A) Stability of the woman's emotional and psychological status B) Degree of glycemic control achieved during the pregnancy C) Evaluation of retinopathy by an ophthalmologist D) Blood urea nitrogen level (BUN. within normal limits

B) Degree of glycemic control achieved during the pregnancy

When assessing cervical effacement of a client in labor, the nurse assesses which of the following characteristics? A) Extent of opening to its widest diameter B) Degree of thinning C) Passage of the mucous plug D) Fetal presenting part

B) Degree of thinning

The nurse administers RhoGAM to an Rh-negative client after delivery of an Rh-positive newborn based on the understanding that this drug will prevent her from: A) Becoming Rh positive B) Developing Rh sensitivity C) Developing AB antigens in her blood D) Becoming pregnant with an Rh-positive fetus

B) Developing Rh sensitivity

Assessment of a newborn reveals uneven gluteal (buttocks. skin creases and a "clunk" when Ortolani's maneuver is performed. Which of the following would the nurse suspect? A) Slipping of the periosteal joint B) Developmental hip dysplasia C) Normal newborn variation D) Overriding of the pelvic bone

B) Developmental hip dysplasia

Which of the following, if noted in the maternal history, would the nurse identify as possibly contributing to the birth of an LGA newborn? A) Drug abuse B) Diabetes C) Preeclampsia D) Infection

B) Diabetes

The nurse prepares to assess a newborn who is considered to be large for gestational age (LGA). Which of the following would the nurse correlate with this gestational age variation? A) Strong, brisk motor skills B) Difficulty in arousing to a quiet alert state C) Birth weight of 7 lb 14 oz D) Wasted appearance of extremities

B) Difficulty in arousing to a quiet alert state

A woman with preterm labor is receiving magnesium sulfate. Which finding would require the nurse to intervene immediately? A) Respiratory rate of 16 breaths per minute B) Diminished deep tendon reflexes C) Urine output of 45 mL/hour D) Alert level of consciousness

B) Diminished deep tendon reflexes

A client is diagnosed with pelvic inflammatory disease (PID). When reviewing the client's medical record, which of the following would the nurse expect to find? (Select all that apply.) A) Oral temperature of 100.4 degrees F B) Dysmenorrhea C) Dysuria D) Lower abdominal tenderness E) Discomfort with cervical motion F) Multiparity

B) Dysmenorrhea C) Dysuria D) Lower abdominal tenderness E) Discomfort with cervical motion

Which finding would the nurse expect to find in a client with endometriosis? A) Hot flashes B) Dysuria C) Fluid retention D) Fever

B) Dysuria

A nurse is developing a teaching plan about nutrition for a group of pregnant women. Which of the following would the nurse include in the discussion? (Select all that apply.) A) Keep weight gain to 15 lb B) Eat three meals with snacking C) Limit the use of salt in cooking D) Avoid using diuretics E) Participate in physical activity

B) Eat three meals with snacking D) Avoid using diuretics E) Participate in physical activity

A nurse is reviewing the medical record of a client. Which of the following would lead the nurse to suspect that the client is experiencing polycystic ovarian syndrome? (Select all that apply.. A) Decreased androgen levels B) Elevated blood insulin levels C) Anovulation D) Waist circumference of 32 inches E) Triglyceride level of 175 mg/dL F) High-density lipoprotein level of 40 mg/dL

B) Elevated blood insulin levels C) Anovulation E) Triglyceride level of 175 mg/dL

A nurse is making a home visit to a postpartum woman who delivered a healthy newborn 4 days ago. The woman's breasts are swollen, hard, and tender to the touch. The nurse documents this finding as which of the following? A) Involution B) Engorgement C) Mastitis D) Engrossment

B) Engorgement

During a prenatal class for a group of new mothers, the nurse is describing the hormones produced by the placenta. Which of the following would the nurse include? (Select all that apply.) A) Prolactin B) Estriol C) Relaxin D) Progestin E) Human chorionic somatomammotropin

B) Estriol C) Relaxin D) Progestin E) Human chorionic somatomammotropin

When teaching a group of postmenopausal women about hot flashes and night sweats, the nurse would address which of the following as the primary cause? A) Poor dietary intake B) Estrogen deficiency C) Active lifestyle D) Changes in vaginal pH

B) Estrogen deficiency

After the nurse describes fetal circulation to a pregnant woman, the woman asks why her fetus has a different circulation pattern than hers. In planning a response, the nurse integrates understanding of which of the following? A) Fetal blood is thicker than that of adults and needs different pathways. B) Fetal circulation carries highly oxygenated blood to vital areas first. C) Fetal blood has a higher oxygen saturation and circulates more slowly. D) Fetal heart rates are rapid and circulation time is double that of adults.

B) Fetal circulation carries highly oxygenated blood to vital areas first.

A primigravida whose labor was initially progressing normally is now experiencing a decrease in the frequency and intensity of her contractions. The nurse would assess the woman for which condition? A) A low-lying placenta B) Fetopelvic disproportion C) Contraction ring D) Uterine bleeding

B) Fetopelvic disproportion

Which findings would the nurse expect to find in a client with bacterial vaginosis? A) Vaginal pH of 3 B) Fish-like odor of discharge C) Yellowish-green discharge D) Cervical bleeding on contact

B) Fish-like odor of discharge

When obtaining the health history from a client, which factor would lead the nurse to suspect that the client has an increased risk for sexually transmitted infections (STIs)? A) Hive-like rash for the past 2 days B) Five different sexual partners C) Weight gain of 5 lbs in 1 year D) Clear vaginal discharge

B) Five different sexual partners

When describing the structures involved in fetal circulation, the nursing instructor describes which structure as the opening between the right and left atrium? A) Ductus venosus B) Foramen ovale C) Ductus arteriosus D) Umbilical artery

B) Foramen ovale

After a normal labor and birth, a client is discharged from the hospital 12 hours later. When the community health nurse makes a home visit 2 days later, which finding would alert the nurse to the need for further intervention? A) Presence of lochia serosa B) Frequent scant voidings C) Fundus firm, below umbilicus D) Milk filling in both breasts

B) Frequent scant voidings

A nurse is completing the assessment of a woman admitted to the labor and birth suite. Which of the following would the nurse expect to include as part of the physical assessment? (Select all that apply.) A) Current pregnancy history B) Fundal height measurement C) Support system D) Estimated date of birth E) Membrane status F) Contraction pattern

B) Fundal height measurement E) Membrane status F) Contraction pattern

A nursing student is preparing a class presentation about changes in the various body systems during the postpartum period and their effects. Which of the following would the student include as influencing a postpartum woman's ability to void? (Select all that apply.) A) Use of an opioid anesthetic during labor B) Generalized swelling of the perineum C) Decreased bladder tone from regional anesthesia D) Use of oxytocin to augment labor

B) Generalized swelling of the perineum C) Decreased bladder tone from regional anesthesia D) Use of oxytocin to augment labo

The nurse would be alert for possible placental abruption during labor when assessment reveals which of the following? A) Macrosomia B) Gestational hypertension C) Gestational diabetes D) Low parity

B) Gestational hypertension

A group of students are reviewing information about sexually transmitted infections and their effect on pregnancy. The students demonstrate understanding of the information when they identify which infection as being responsible for ophthalmia neonatorum? A) Syphilis B) Gonorrhea C) Chlamydia D) HPV

B) Gonorrhea

A client is admitted to the labor and birthing suite in early labor. On review of her medical record, the nurse determines that the client's pelvic shape as identified in the antepartal progress notes is the most favorable one for a vaginal delivery. Which pelvic shape would the nurse have noted? A) Platypelloid B) Gynecoid C) Android D) Anthropoid

B) Gynecoid

A newborn is scheduled to undergo a screening test for phenylketonuria (PKU). The nurse prepares to obtain the blood sample from the newborn's: A) Finger B) Heel C) Scalp vein D) Umbilical vein

B) Heel

A nurse is massaging a postpartum client's fundus and places the nondominant hand on the area above the symphysis pubis based on the understanding that this action: A) Determines that the procedure is effective B) Helps support the lower uterine segment C) Aids in expressing accumulated clots D) Prevents uterine muscle fatigue

B) Helps support the lower uterine segment

The nurse is reviewing the laboratory test results of a client with dysfunctional uterine bleeding (DUB). Which finding would be of concern? A) Negative pregnancy test B) Hemoglobin level of 10.1 g/dL C) Prothrombin time of 60 seconds D) Serum cholesterol of 140 mg/dL

B) Hemoglobin level of 10.1 g/dL

When developing a teaching plan for a community group about HIV infection, which group would the nurse identify as an emerging risk group for HIV infection? A) Native Americans B) Heterosexual women C) New health care workers D) Asian immigrants

B) Heterosexual women

The nurse is assessing a postpartum client's lochia and finds that there is about a 4-inch stain on the perineal pad. The nurse documents this finding as which of the following? A) Scant B) Light C) Moderate D) Large

B) Light

A nurse is observing a postpartum woman and her partner interact with the their newborn. The nurse determines that the parents are developing parental attachment with their newborn when they demonstrate which of the following? (Select all that apply.) A) Frequently ask for the newborn to be taken from the room B) Identify common features between themselves and the newborn C) Refer to the newborn as having a monkey-face D) Make direct eye contact with the newborn E) Refrain from checking out the newborn's features

B) Identify common features between themselves and the newborn, D) Make direct eye contact with the newborn

Which of the following would the nurse interpret as being least indicative of paternal engrossment? A) Demonstrating pleasure when touching or holding the newborn B) Identifying imperfections in the newborn's appearance C) Being able to distinguish his newborn from others in the nursery D) Showing feelings of pride with the birth of the newborn

B) Identifying imperfections in the newborn's appearance

During a routine prenatal visit, a client, 36 weeks pregnant, states she has difficulty breathing and feels like her pulse rate is really fast. The nurse finds her pulse to be 100 beats per minute (increased from baseline readings of 70 to 74 beats per minute. and irregular, with bilateral crackles in the lower lung bases. Which nursing diagnosis would be the priority for this client? A) Ineffective tissue perfusion related to supine hypotensive syndrome B) Impaired gas exchange related to pulmonary congestion C) Activity intolerance related to increased metabolic requirements D) Anxiety related to fear of pregnancy outcome

B) Impaired gas exchange related to pulmonary congestion

A mother brings her 12-year-old daughter in for well-visit checkup. During the visit, the nurse is discussing the use of prophylactic HPV vaccine for the daughter. The mother agrees and the daughter receives her first dose. The nurse schedules the daughter for the next dose, which would be given at which time? A) In 2 month B) In 2 months C) In 3 months D) In 4 months

B) In 2 months

A nurse is describing the risks associated with prolonged pregnancies as part of an inservice presentation. Which of the following would the nurse be least likely to incorporate in the discussion as an underlying reason for problems in the fetus? A) Aging of the placenta B) Increased amniotic fluid volume C) Meconium aspiration D) Cord compression

B) Increased amniotic fluid volume

A nursing instructor is preparing a teaching plan for a group of nursing students about the potential for misuse of genetic discoveries and advances. Which the following would the instructor most likely include? A) Gene replacement therapy for defective genes B) Individual risk profiling and confidentiality C) Greater emphasis on the causes of diseases D) Slower diagnosis of specific diseases

B) Individual risk profiling and confidentiality

After presenting a class on measures to prevent postpartum hemorrhage, the presenter determines that the teaching was successful when the class states which of the following as an important measure to prevent postpartum hemorrhage due to retained placental fragments? A) Administering broad-spectrum antibiotics B) Inspecting the placenta after delivery for intactness C) Manually removing the placenta at delivery D) Applying pressure to the umbilical cord to remove the placenta

B) Inspecting the placenta after delivery for intactness

A nurse is assessing a newborn who is about 4½ hours old. The nurse would expect this newborn to exhibit which of the following? (Select all that apply.) A) Sleeping B) Interest in environmental stimuli C) Passage of meconium D) Difficulty arousing the newborn E) Spontaneous Moro reflexes

B) Interest in environmental stimuli C) Passage of meconium

A client's last menstrual period was April 11. Using Nagele's rule, her expected date of birth (EDB. would be: A) January 4 B) January 18 C) January 25 D) February 24

B) January 18

During a wellness visit to the clinic, a woman asks the nurse if there is anything she can do to reduce her risk for developing breast cancer. Which of the following would the nurse most likely include? (Select all that apply.) A) Eating three servings of fruit daily B) Keeping weight gain under 11 pounds after age 18 C) Eating at least seven portions of complex carbohydrates daily D) Limiting the intake of refined sugar products E) Using salt liberally when cooking

B) Keeping weight gain under 11 pounds after age 18 C) Eating at least seven portions of complex carbohydrates daily D) Limiting the intake of refined sugar products

A group of nursing students are reviewing information about methods used for cervical ripening. The students demonstrate understanding of the information when they identify which of the following as a mechanical method? A) Herbal agents B) Laminaria C) Membrane stripping D) Amniotomy

B) Laminaria

The nurse is reviewing the medical record of a newborn born 2 hours ago. The nurse notes that the newborn was delivered at 35 weeks' gestation. The nurse would classify this newborn as which of the following? A) Preterm B) Late preterm C) Full term D) Postterm

B) Late preterm

The nurse is auscultating a newborn's heart and places the stethoscope at the point of maximal impulse at which location? A) Just superior to the nipple, at the midsternum B) Lateral to the midclavicular line at the fourth intercostal space C) At the fifth intercostal space to the left of the sternum D) Directly adjacent to the sternum at the second intercostals space

B) Lateral to the midclavicular line at the fourth intercostal space

A woman in her third trimester comes to the clinic for a prenatal visit. During assessment the woman reports that her breathing has become much easier in the last week but she has noticed increased pelvic pressure , cramping and lower back pain. The nurse determines that which of the following has most likely occurred? A) Cervical dilation B) Lightening C) Bloody show D) Braxton-Hicks contractions

B) Lightening

A nurse is assessing a child with Klinefelter's syndrome. Which of the following would the nurse expect to assess? (Select all that apply.) A) Gross mental retardation B) Long arms C) Profuse body hair D) Gynecomastia E) Enlarged testicles

B) Long arms D) Gynecomastia

When describing newborns with birth-weight variations to a group of nursing students, the instructor identifies which variation if the newborn weighs 5.2 lb at any gestational age? A) Small for gestational age B) Low birth weight C) Very low birth weight D) Extremely low birth weight

B) Low birth weight

The nurse is developing a plan of care for a woman with breast cancer who is scheduled to undergo breast-conserving surgery. The nurse interprets this as which of the following? A) Removal of nipple and areolar area B) Lump removal followed by radiation C) Entire breast removal without lymph nodes D) Axillary lymph node removal

B) Lump removal followed by radiation

When developing the plan of care for a woman who has had an abdominal hysterectomy, which of the following would be contraindicated? A) Ambulating the client B) Massaging the client's legs C) Applying elasticized stockings D) Encouraging range-of-motion exercises

B) Massaging the client's legs

A nurse strongly encourages a pregnant client to avoid eating swordfish and tilefish because these fish contain which of the following? A) Excess folic acid, which could increase the risk for neural tube defects B) Mercury, which could harm the developing fetus if eaten in large amounts C) Lactose, which leads to abdominal discomfort, gas, and diarrhea D) Low-quality protein that does not meet the woman's requirements

B) Mercury, which could harm the developing fetus if eaten in large amounts

A postpartum client comes to the clinic for her routine 6-week visit. The nurse assesses the client and suspects that she is experiencing subinvolution based on which of the following? A) Nonpalpable fundus B) Moderate lochia serosa C) Bruising on arms and legs D) Fever

B) Moderate lochia serosa

A woman calls the health care facility stating that she is in labor. The nurse would urge the client to come to the facility if the client reports which of the following? A) Increased energy level with alternating strong and weak contractions B) Moderately strong contractions every 4 minutes, lasting about 1 minute C) Contractions noted in the front of abdomen that stop when she walks D) Pink-tinged vaginal secretions and irregular contractions lasting about 30 seconds

B) Moderately strong contractions every 4 minutes, lasting about 1 minute

During a physical assessment of a newborn, the nurse observes bluish markings across the newborn's lower back. The nurse documents this finding as which of the following? A) Milia B) Mongolian spots C) Stork bites D) Birth trauma

B) Mongolian spots

A nurse is observing a postpartum client interacting with her newborn and notes that the mother is engaging with the newborn in the en face position. Which of the following would the nurse be observing? A) Mother placing the newborn next to bare breast. B) Mother making eye-to-eye contact with the newborn C) Mother gently stroking the newborn's face D) Mother holding the newborn upright at the shoulder

B) Mother making eye-to-eye contact with the newborn

Assessment of a newborn reveals rhythmic spontaneous movements. The nurse interprets this as indicating: A) Habituation B) Motor maturity C) Orientation D) Social behaviors

B) Motor maturity

A nurse is assessing a woman after birth and notes a second-degree laceration. The nurse interprets this as indicating that the tear extends through which of the following? A) Skin B) Muscles of perineal body C) Anal sphincter D) Anterior rectal wall

B) Muscles of perineal body

A nurse is teaching a class on X-linked recessive disorders. Which of the following statements would the nurse most likely include? A) Males are typically carriers of the disorders. B) No male-to-male transmission occurs. C) Daughters are more commonly affected with the disorder. D) Both sons and daughters have a 50% risk of the disorder.

B) No male-to-male transmission occurs.

After teaching a group of nursing students about a neutral thermal environment, the instructor determines that the teaching was successful when the students identify which of the following as the newborn's primary method of heat production? A) Convection B) Nonshivering thermogenesis C) Cold stress D) Bilirubin conjugation

B) Nonshivering thermogenesis

A nurse is working as part of a committee to establish policies to promote bonding and attachment. Which practice would be least effective in achieving this goal? A) Allowing unlimited visiting hours on maternity units B) Offering round-the-clock nursery care for all infants C) Promoting rooming-in D) Encouraging infant contact immediately after birth

B) Offering round-the-clock nursery care for all infants

When assessing several women for possible VBAC, which woman would the nurse identify as being the best candidate? A) One who has undergone a previous myomectomy B) One who had a previous cesarean birth via a low transverse incision C) One who has a history of a contracted pelvis D) One who has a vertical incision from a previous cesarean birth

B) One who had a previous cesarean birth via a low transverse incision

A client's maternal serum alpha-fetoprotein (MSAFP. level was unusually elevated at 17 weeks. The nurse suspects which of the following? A) Fetal hypoxia B) Open spinal defects C) Down syndrome D) Maternal hypertension

B) Open spinal defects

A woman has opted to use the basal body temperature method for contraception. The nurse instructs the client that a rise in basal body temperature indicates which of the following? A) Onset of menses B) Ovulation C) Pregnancy D) Safe period for intercourse

B) Ovulation

The nurse prepares to administer a gavage feeding for a newborn with transient tachypnea based on the understanding that this type of feeding is necessary for which reason? A) Lactase enzymatic activity is not adequate. B) Oxygen demands need to be reduced. C) Renal solute lead must be considered. D) Hyperbilirubinemia is likely to develop.

B) Oxygen demands need to be reduced.

A woman with gestational hypertension experiences a seizure. Which of the following would be the priority? A) Fluid replacement B) Oxygenation C) Control of hypertension D) Delivery of the fetus

B) Oxygenation

The fetus of a woman in labor is determined to be in persistent occiput posterior position. Which of the following would the nurse identify as the priority intervention? A) Position changes B) Pain relief measures C) Immediate cesarean birth D) Oxytocin administration

B) Pain relief measures

After teaching a group of students about the different perinatal education methods, the instructor determines that the teaching was successful when the students identify which of the following as the Bradley method? A) Psychoprophylactic method B) Partner-coached method C) Natural childbirth method D) Mind prevention method

B) Partner-coached method

Which of the following would the nurse include when teaching a new mother about the difference between pathologic and physiologic jaundice? A) Physiologic jaundice results in kernicterus. B) Pathologic jaundice appears within 24 hours after birth. C) Both are treated with exchange transfusions of maternal O- blood. D) Physiologic jaundice requires transfer to the NICU.

B) Pathologic jaundice appears within 24 hours after birth

A group of students are reviewing information about genetic inheritance. The students demonstrate understanding of the information when they identify which of the following as an example of an autosomal recessive disorder? (Select all that apply.) A) Cystic fibrosis B) Phenylketonuria C) Tay-Sachs disease D) Polycystic kidney disease E) Achondroplasia

B) Phenylketonuria D) Polycystic kidney disease

Assessment of a pregnant woman reveals that she compulsively craves ice. The nurse documents this finding as which of the following? A) Quickening B) Pica C) Ballottement D) Linea nigra

B) Pica

A nurse is visiting a postpartum woman who delivered a healthy newborn 5 days ago. Which of the following would the nurse expect to find? A) Bright red discharge B) Pinkish brown discharge C) Deep red mucus-like discharge D) Creamy white discharge

B) Pinkish brown discharge

A pregnant woman diagnosed with syphilis comes to the clinic for a visit. The nurse discusses the risk of transmitting the infection to her newborn, explaining that this infection is transmitted to the newborn through the: A) Amniotic fluid B) Placenta C) Birth canal D) Breast milk

B) Placenta

After teaching a group of students about risk factors associated with postpartum hemorrhage, the instructor determines that the teaching was successful when the students identify which of the following as a risk factor? (Select all that apply.) A) Prolonged labor B) Placenta previa C) Null parity D) Hydramnios E) Labor augmentation

B) Placenta previa, D) Hydramnios, E) Labor augmentation

Assessment of a pregnant woman reveals oligohydramnios. The nurse would be alert for the development of which of the following? A) Maternal diabetes B) Placental insufficiency C) Neural tube defects D) Fetal gastrointestinal malformations

B) Placental insufficiency

A nurse is reviewing the laboratory test results of a newborn. Which result would the nurse identify as a cause for concern? A) Hemoglobin 19 g/dL B) Platelets 75,000/uL C) White blood cells 20,000/mm3 D) Hematocrit 52%

B) Platelets 75,000/uL

A nurse is providing care to a female client receiving treatment for a Bartholin's cyst. The client has had a small loop of plastic tubing secured in place to allow for drainage. The nurse instructs the client that she will have a follow-up appointment for removal of the plastic tubing at which time? A) 1 week B) 2 weeks C) 3 weeks D) 4 weeks

C) 3 weeks

A nurse is developing a teaching plan for a postpartum woman who is breast-feeding about sexuality and contraception. Which of the following would the nurse most likely include? (Select all that apply.) A) Resumption of sexual intercourse about two weeks after delivery B) Possible experience of fluctuations in sexual interest C) Use of a water-based lubricant to ease vaginal discomfort D) Use of combined hormonal contraceptives for the first three weeks E) Possibility of increased breast sensitivity during sexual activity

B) Possible experience of fluctuations in sexual interest, C) Use of a water-based lubricant to ease vaginal discomfort, E) Possibility of increased breast sensitivity during sexual activity

A woman's amniotic fluid is noted to be cloudy. The nurse interprets this finding as which of the following? A) Normal B) Possible infection C) Meconium passage D) Transient fetal hypoxia

B) Possible infection

The nurse is developing a plan of care for a woman who is pregnant with twins. The nurse includes interventions focusing on which of the following because of the woman's increased risk? A) Oligohydramnios B) Preeclampsia C) Post-term labor D) Chorioamnionitis

B) Preeclampsia

After teaching a class on the stages of fetal development, the instructor determines that the teaching was successful when the students identify which of the following as a stage? (Select all that apply.) A) Placental B) Preembryonic C) Umbilical D) Embryonic E) Fetal

B) Preembryonic C) Umbilical E) Fetal

A nurse is reviewing the medical record of a pregnant woman and notes that she is gravid II. The nurse interprets this to indicate the number of: A) Deliveries B) Pregnancies C) Spontaneous abortions D) Pre-term births

B) Pregnancies

A newborn is diagnosed with meconium aspiration syndrome. When assessing this newborn, which of the following would the nurse expect to find? (Select all that apply.) A) Pigeon chest B) Prolonged tachypnea C) Intercostal retractions D) High blood pH level E) Coarse crackles on auscultation

B) Prolonged tachypnea C) Intercostal retractions E) Coarse crackles on auscultation

When describing perinatal education to a pregnant woman and her partner, the nurse emphasizes that the primary goal of these classes is to: A) Equip a couple with the knowledge to experience a pain-free childbirth B) Provide knowledge and skills to actively participate in birth and parenting C) Eliminate anxiety so that they can have an uncomplicated birth D) Empower the couple to totally control the birth process

B) Provide knowledge and skills to actively participate in birth and parenting

Which of the following would be most appropriate for the nurse to do when assisting parents who have experienced the loss of their preterm newborn? A) Avoid using the terms "death" or "dying." B) Provide opportunities for them to hold the newborn. C) Refrain from initiating conversations with the parents. D) Quickly refocus the parents to a more pleasant topic.

B) Provide opportunities for them to hold the newborn.

The partner of a woman who has given birth to a healthy newborn says to the nurse, "I want to be involved, but I'm not sure that I'm able to care for such a little baby." The nurse interprets this as indicating which of the following stages? A) Expectations B) Reality C) Transition to mastery D) Taking-hold

B) Reality

A nurse at a local community clinic is developing a program to address STI prevention. Which of the following would the nurse least likely include in the program? A) Outlining safer sexual behavior B) Recommending screening for symptomatic individuals C) Promoting the use of barrier contraceptives D) Offering education about STI transmission

B) Recommending screening for symptomatic individuals

A group of nursing students are reviewing respiratory system adaptations that occur during the postpartum period. The students demonstrate understanding of the information when they identify which of the following as a postpartum adaptation? A) Continued shortness of breath B) Relief of rib aching C) Diaphragmatic elevation D) Decrease in respiratory rate

B) Relief of rib aching

Just after delivery, a newborn's axillary temperature is 94° C. What action would be most appropriate? A) Assess the newborn's gestational age. B) Rewarm the newborn gradually. C) Observe the newborn every hour. D) Notify the physician if the temperature goes lower.

B) Rewarm the newborn gradually.

A group of nursing students are reviewing information about mastitis and its causes. The students demonstrate understanding of the information when they identify which of the following as the most common cause? A) E. coli B) S. aureus C) Proteus D) Klebsiella

B) S. aureus

Assessment of a newborn reveals transient tachypnea. The nurse reviews the newborn's medical record. Which of the following would the nurse be least likely to identify as a risk factor for this condition? A) Cesarean birth B) Shortened labor C) Central nervous system depressant during labor D) Maternal asthma

B) Shortened labor

When making a home visit, the nurse observes a newborn sleeping on his back in a bassinet. In one corner of the bassinet is a soft stuffed animal and at the other end is a bulb syringe. The nurse determines that the mother needs additional teaching because of which of the following? A) The newborn should not be sleeping on his back. B) Stuffed animals should not be in areas where infants sleep. C) The bulb syringe should not be kept in the bassinet. D) This newborn should be sleeping in a crib.

B) Stuffed animals should not be in areas where infants sleep.

A laboratory technician arrives to draw blood for a complete blood count (CBC. for a client who had a right-sided mastectomy 8 hours ago. The client has an intravenous line with fluid infusing in her left antecubital space. To obtain the blood specimen, the technician places a tourniquet on the client's right arm. Which action by the nurse would be most appropriate? A) Assist in holding the client's arm still. B) Suggest a finger stick be done on one of the client's left fingers. C) Tell the technician to obtain the blood sample from the client's left arm. D) Call the surgeon to perform a femoral puncture.

B) Suggest a finger stick be done on one of the client's left fingers.

A client with advanced breast cancer, who has had both chemotherapy and radiation therapy, is to start hormonal therapy. Which agent would the nurse expect the client to receive? A) Progestins B) Tamoxifen C) Cortisone D) Estrogen

B) Tamoxifen

A group of nursing students are reviewing the different types of congenital heart disease in infants. The students demonstrate a need for additional review when they identify which of the following as an example of increased pulmonary blood flow (left-to-right shunting)? A) Atrial septal defect B) Tetralogy of Fallot C) Ventricular septal defect D) Patent ductus arteriosus

B) Tetralogy of Fallot

After teaching a group of students about the maternal bony pelvis, which statement by the group indicates that the teaching was successful? A) The bony pelvis plays a lesser role during labor than soft tissue. B) The pelvic outlet is associated with the true pelvis. C) The false pelvis lies below the imaginary linea terminalis. D) The false pelvis is the passageway through which the fetus travels.

B) The pelvic outlet is associated with the true pelvis.

The nurse is assessing a newborn of a woman who is suspected of abusing alcohol. Which newborn finding would provide additional evidence to support this suspicion? A) Wide large eyes B) Thin upper lip C) Protruding jaw D) Elongated nose

B) Thin upper lip

A father of a newborn tells the nurse, "I may not know everything about being a dad, but I'm going to do the best I can for my son." The nurse interprets this as indicating the father is in which stage of adaptation? A) Expectations B) Transition to mastery C) Reality D) Taking-in

B) Transition to mastery

A primiparous client is being seen in the clinic for her first prenatal visit. It is determined that she is 11 weeks pregnant. The nurse develops a teaching plan to educate the client about what she will most likely experience during this period. Which of the following would the nurse include? A) Ankle edema B) Urinary frequency C) Backache D) Hemorrhoids

B) Urinary frequency

A nursing student is reviewing an article about preterm premature rupture of membranes. Which of the following would the student expect to find as factor placing a woman at high risk for this condition? (Select all that apply.) A) High body mass index B) Urinary tract infection C) Low socioeconomic status D) Single gestations E) Smoking

B) Urinary tract infection C) Low socioeconomic status E) Smoking

A group of students are reviewing the causes of postpartum hemorrhage. The students demonstrate understanding of the information when they identify which of the following as the most common cause? A) Labor augmentation B) Uterine atony C) Cervical or vaginal lacerations D) Uterine inversion

B) Uterine atony

A pregnant woman comes to the clinic and tells the nurse that she has been having a whitish vaginal discharge. The nurse suspects vulvovaginal candidiasis based on which assessment finding? A) Fever B) Vaginal itching C) Urinary frequency D) Incontinence

B) Vaginal itching

A nurse is reviewing the fetal heart rate pattern and observes abrupt decreases in FHR below the baseline, appearing as a U-shape. The nurse interprets these changes as reflecting which of the following? A) Early decelerations B) Variable decelerations C) Prolonged decelerations D) Late decelerations

B) Variable decelerations

When planning the care of a newborn addicted to cocaine who is experiencing withdrawal, which of the following would be least appropriate to include? A) Wrapping the newborn snugly in a blanket B) Waking the newborn every hour C) Checking the newborn's fontanels D) Offering a pacifier

B) Waking the newborn every hour

A nursing student is preparing a presentation on minimizing heat loss in the newborn. Which of the following would the student include as a measure to prevent heat loss through convection? A) Placing a cap on a newborn's head B) Working inside an isolette as much as possible. C) Placing the newborn skin-to-skin with the mother D) Using a radiant warmer to transport a newborn

B) Working inside an isolette as much as possible.

The nurse observes the stool of a newborn who has begun to breast-feed. Which of the following would the nurse expect to find? A) Greenish black, tarry stool B) Yellowish-brown, seedy stool C) Yellow-gold, stringy stool D) Yellowish-green, pasty stool

B) Yellowish-brown, seedy stool

A nursing instructor is teaching a class to a group of students about pregnancy, insulin, and glucose. Which of the following would the instructor least likely include as opposing insulin? A) Prolactin B) Estrogen C) Progesterone D) Cortisol

D) Cortisol

The nurse is discussing the insulin needs of a primaparous client with diabetes who has been using insulin for the past few years. The nurse informs the client that her insulin needs will increase during pregnancy based on the nurse's understanding that the placenta produces: A) hCG, which increases maternal glucose levels B) hPL, which deceases the effectiveness of insulin C) Estriol, which interferes with insulin crossing the placenta D) Relaxin, which decreases the amount of insulin produced

B) hPL, which deceases the effectiveness of insulin

A primipara client who is bottle feeding her baby begins to experience breast engorgement on her third postpartum day. Which instruction would be most appropriate to aid in relieving her discomfort? A) "Express some milk from your breasts every so often to relieve the distention." B) "Remove your bra to relieve the pressure on your sensitive nipples and breasts." C) "Apply ice packs to your breasts to reduce the amount of milk being produced." D) "Take several warm showers daily to stimulate the milk let-down reflex."

C) "Apply ice packs to your breasts to reduce the amount of milk being produced."

A gravida 2 para 1 client in the 10th week of her pregnancy says to the nurse, "I've never urinated as often as I have for the past three weeks." Which response would be most appropriate for the nurse to make? A) "Having to urinate so often is annoying. I suggest that you watch how much fluid you are drinking and limit it." B) "You shouldn't be urinating this frequently now; it usually stops by the time you're eight weeks pregnant. Is there anything else bothering you?" C) "By the time you are 12 weeks pregnant, this frequent urination should no longer be a problem, but it is likely to return toward the end of your pregnancy." D) "Women having their second child generally don't have frequent urination. Are you experiencing any burning sensations?"

C) "By the time you are 12 weeks pregnant, this frequent urination should no longer be a problem, but it is likely to return toward the end of your pregnancy."

A woman with hyperemesis gravidarum asks the nurse about suggestions to minimize nausea and vomiting. Which suggestion would be most appropriate for the nurse to make? A) "Make sure that anything around your waist is quite snug." B) "Try to eat three large meals a day with less snacking." C) "Drink fluids in between meals rather than with meals." D) "Lie down for about an hour after you eat"

C) "Drink fluids in between meals rather than with meals."

A nurse is working with a pregnancy woman to schedule follow-up visits for her pregnancy. Which statement by the woman indicates that she understands the scheduling? A) "I need to make visits every 2 months until I'm 36 weeks pregnant." B) "Once I get to 28 weeks, I have to come twice a month." C) "From now until I'm 28 weeks, I'll be coming once a month." D) "I'll make sure to get a day off every 2 weeks to make my visits."

C) "From now until I'm 28 weeks, I'll be coming once a month."

A new mother who is breast-feeding her newborn asks the nurse, "How will I know if my baby is drinking enough?" Which response by the nurse would be most appropriate? A) "If he seems content after feeding, that should be a sign." B) "Make sure he drinks at least 5 minutes on each breast." C) "He should wet between 6 to 12 diapers each day." D) "If his lips are moist, then he's okay."

C) "He should wet between 6 to 12 diapers each day."

A couple comes to the clinic for a fertility evaluation. The male partner is to undergo a semen analysis. After teaching the partner about this test, which client statement indicates that the client has understood the instructions? A) "I need to bring the specimen to the lab the day after collecting it." B) "I will place the specimen in a special plastic bag to transport it." C) "I have to abstain from sexual activity for about 1-2 days before the sample." D) "I will withdraw before I ejaculate during sex to collect the specimen."

C) "I have to abstain from sexual activity for about 1-2 days before the sample."

After teaching a pregnant woman with iron deficiency anemia about her prescribed iron supplement, which statement indicates successful teaching? A) "I should take my iron with milk." B) "I should avoid drinking orange juice." C) "I need to eat foods high in fiber." D) "I'll call the doctor if my stool is black and tarry."

C) "I need to eat foods high in fiber."

A pregnant woman is flying across the country to visit her family. After teaching the woman about traveling during pregnancy, which statement indicates that the teaching was successful? A) "I'll sit in a window seat so I can focus on the sky to help relax me." B) "I won't drink too much fluid so I don't have to urinate so often." C) "I'll get up and walk around the airplane about every 2 hours." D) "I'll do some upper arm stretches while sitting in my seat."

C) "I'll get up and walk around the airplane about every 2 hours."

Upon entering the room of a client who has had a spontaneous abortion, the nurse observes the client crying. Which of the following responses by the nurse would be most appropriate? A) "Why are you crying?" B) "Will a pill help your pain?" C) "I'm sorry you lost your baby." D) "A baby still wasn't formed in your uterus."

C) "I'm sorry you lost your baby."

A nurse is teaching a pregnant woman with preterm premature rupture of membranes who is about to be discharged home about caring for herself. Which statement by the woman indicates a need for additional teaching? A) "I need to keep a close eye on how active my baby is each day." B) "I need to call my doctor if my temperature increases." C) "It's okay for my husband and me to have sexual intercourse." D) "I can shower but I shouldn't take a tub bath."

C) "It's okay for my husband and me to have sexual intercourse."

A woman in her second trimester comes for a follow-up visit and says to the nurse, "I feel like I'm on an emotional roller-coaster." Which response by the nurse would be most appropriate? A) "How often has this been happening to you?" B) "Maybe you need some medication to level things out." C) "Mood swings are completely normal during pregnancy." D) "Have you been experiencing any thoughts of harming yourself?"

C) "Mood swings are completely normal during pregnancy."

The nurse is teaching a pregnant woman with type 1 diabetes about her diet during pregnancy. Which client statement indicates that the nurse's teaching was successful? A) "I'll basically follow the same diet that I was following before I became pregnant." B) "Because I need extra protein, I'll have to increase my intake of milk and meat." C) "Pregnancy affects insulin production, so I'll need to make adjustments in my diet." D) "I'll adjust my diet and insulin based on the results of my urine tests for glucose."

C) "Pregnancy affects insulin production, so I'll need to make adjustments in my diet."

A woman in her 40th week of pregnancy calls the nurse at the clinic and says she's not sure whether she is in true or false labor. Which statement by the client would lead the nurse to suspect that the woman is experiencing false labor? A) "I'm feeling contractions mostly in my back." B) "My contractions are about 6 minutes apart and regular." C) "The contractions slow down when I walk around." D) "If I try to talk to my partner during a contraction, I can't."

C) "The contractions slow down when I walk around."

A client with genital herpes simplex infection asks the nurse, "Will I ever be cured of this infection?" Which response by the nurse would be most appropriate? A) "There is a new vaccine available that prevents the infection from returning." B) "All you need is a dose of penicillin and the infection will be gone." C) "There is no cure, but drug therapy helps to reduce symptoms and recurrences." D) "Once you have the infection, you develop an immunity to it."

C) "There is no cure, but drug therapy helps to reduce symptoms and recurrences."

When describing amniotic fluid to a pregnant woman, the nurse would include which of the following? A) "This fluid acts as transport mechanism for oxygen and nutrients." B) "The fluid is mostly protein to provide nourishment to your baby." C) "This fluid acts as a cushion to help to protect your baby from injury." D) "The amount of fluid remains fairly constant throughout the pregnancy."

C) "This fluid acts as a cushion to help to protect your baby from injury."

A nurse is explaining the use of therapeutic touch as a pain relief measure during labor. Which of the following would the nurse include in the explanation? A) "This technique focuses on manipulating body tissues." B) "The technique requires focusing on a specific stimulus." C) "This technique redirects energy fields that lead to pain." D) "The technique involves light stroking of the abdomen with breathing."

C) "This technique redirects energy fields that lead to pain."

A pregnant woman in the 36th week of gestation complains that her feet are quite swollen at the end of the day. After careful assessment, the nurse determines that this is an expected finding at this stage of pregnancy. Which intervention would be most appropriate for the nurse to suggest? A) "Limit your intake of fluids." B) "Eliminate salt from your diet." C) "Try elevating your legs when you sit." D) "Wear Spandex-type full-length pants."

C) "Try elevating your legs when you sit."

The nurse is developing the discharge plan for a woman who has had a left-sided modified radical mastectomy. The nurse is including instructions for ways to minimize lymphedema. Which suggestion would most likely increase the woman's symptoms? A) "Wear gloves when you are doing any gardening." B) "Have your blood pressure taken in your right arm." C) "Wear clothing with elasticized sleeves." D) "Avoid driving to and from work every day."

C) "Wear clothing with elasticized sleeves."

After teaching a group of nursing students about the impact of pregnancy on the older woman, the instructor determines that the teaching was successful when the students state which of the following? A) "The majority of women who become pregnant over age 35 experience complications." B) "Women over the age of 35 who become pregnant require a specialized type of assessment." C) "Women over age 35 and are pregnant have an increased risk for spontaneous abortions." D) "Women over age 35 are more likely to have substance abuse problems."

C) "Women over age 35 and are pregnant have an increased risk for spontaneous abortions."

A woman is admitted to the labor and birthing suite. Vaginal examination reveals that the presenting part is approximately 2 cm above the ischial spines. The nurse documents this finding as: A) +2 station B) 0 station C) -2 station D) Crowning

C) -2 station

Assessment of a pregnant woman reveals that the presenting part of the fetus is at the level of the maternal ischial spines. The nurse documents this as which station? A) -2 B) -1 C) 0 D) +1

C) 0

A nurse measures a pregnant woman's fundal height and finds it to be 28 cm. The nurse interprets this to indicate which of the following? A) 14 weeks' gestation B) 20 weeks' gestation C) 28 weeks' gestation D) 36 weeks' gestation

C) 28 weeks' gestation

A client who is 4 months pregnant is at the prenatal clinic for her initial visit. Her history reveals she has 7-year-old twins who were born at 34 weeks gestation, a 2-year old son born at 39 weeks gestation, and a spontaneous abortion 1 year ago at 6 weeks gestation. Using the GTPAL method, the nurse would document her obstetric history as: A) 3 2 1 0 3 B) 3 1 2 2 3 C) 4 1 1 1 3 D) 4 2 1 3 1

C) 4 1 1 1 3

A woman comes to the clinic and asks the nurse about when she should have her first mammogram. Using the recommendations of the American Cancer Society, which would the nurse suggest? A) 30 years B) 35 years C) 40 years D) 45 years

C) 40 years

The nurse is reviewing the medical records of several clients. Which client would the nurse expect to have an increased risk for developing osteoporosis? A) A woman of African American descent B) A woman who plays tennis twice a week C) A thin woman with small bones D) A woman who drinks one cup of coffee a day

C) A thin woman with small bones

The nurse is assessing a newborn who is large for gestational age. The newborn was born breech. The nurse suspects that the newborn may have experienced trauma to the upper brachial plexus based on which assessment findings? A) Absent grasp reflex B) Hand weakness C) Absent Moro reflex D) Facial asymmetry

C) Absent Moro reflex

A nurse is developing a plan of care for a preterm infant experiencing respiratory distress. Which of the following would the nurse be least likely to include in this plan? A) Stimulate the infant with frequent handling. B) Keep the newborn in a warmed isolette. C) Administer oxygen using a oxygen hood. D) Give gavage or continous tube feedings.

C) Administer oxygen using a oxygen hood.

The nurse is assessing a woman with abruption placentae who has just given birth. The nurse would be alert for which of the following? A) Severe uterine pain B) Board-like abdomen C) Appearance of petechiae D) Inversion of the uterus

C) Appearance of petechiae

To decrease the pain associated with an episiotomy immediately after birth, which action by the nurse would be most appropriate? A) Offer warm blankets. B) Encourage the woman to void. C) Apply an ice pack to the site. D) Offer a warm sitz bath.

C) Apply an ice pack to the site.

A group of pregnant women are discussing high-risk newborn conditions as part of a prenatal class. When describing the complications that can occur in these newborns to the group, which would the nurse include as being at lowest risk? A) Small-for-gestational-age (SGA. newborns B) Large-for-gestational-age (LGA. newborns C) Appropriate-for-gestational-age (AGA. newborns D) Low-birth-weight newborns

C) Appropriate-for-gestational-age (AGA. newborns

A nurse is assessing a postpartum woman's adjustment to her maternal role. Which of the following would the nurse expect to occur first? A) Reestablishing relationships with others B) Demonstrating increasing confidence in care of the newborn C) Assuming a passive role in meeting her own needs D) Becoming preoccupied with the present

C) Assuming a passive role in meeting her own needs

A woman is at 20 weeks' gestation. The nurse would expect to find the fundus at which of the following? A) Just above the symphysis pubis B) Mid-way between the pubis and umbilicus C) At the level of the umbilicus D) Mid-way between the umbilicus and xiphoid process

C) At the level of the umbilicus

A group of nursing students are reviewing information about maternal and paternal adaptations to the birth of a newborn. The nurse observes the parents interacting with their newborn physically and emotionally. The nurse documents this as which of the following? A) Puerperium B) Lactation C) Attachment D) Engrossment

C) Attachment

When applying the ultrasound transducers for continuous external electronic fetal monitoring, at which location would the nurse place the transducer to record the FHR? A) Over the uterine fundus where contractions are most intense B) Above the umbilicus toward the right side of the diaphragm C) Between the umbilicus and the symphysis pubis D) Between the xiphoid process and umbilicus

C) Between the umbilicus and the symphysis pubis

After teaching parents about their newborn, the nurse determines that the teaching was successful when they identify the development of a close emotional attraction to a newborn by parents during the first 30 to 60 minutes after birth as which of the following? A) Reciprocity B) Engrossment C) Bonding D) Attachment

C) Bonding

The nurse is assessing a preterm newborn's fluid and hydration status. Which of the following would alert the nurse to possible overhydration? A) Decreased urine output B) Tachypnea C) Bulging fontanels D) Elevated temperature

C) Bulging fontanels

A client is diagnosed with an enterocele. The nurse interprets this condition as: A) Protrusion of the posterior bladder wall downward through the anterior vaginal wall B) Sagging of the rectum with pressure exerted against the posterior vaginal wall C) Bulging of the small intestine through the posterior vaginal wall D) Descent of the uterus through the pelvic floor into the vagina

C) Bulging of the small intestine through the posterior vaginal wall

The nurse is reviewing the medical record of a woman in labor and notes that the fetal position is documented as LSA. The nurse interprets this information as indicating which of the following is the presenting part? A) Occiput B) Face C) Buttocks D) Shoulder

C) Buttocks

Assessment of a female client reveals a thick, white vaginal discharge. She also reports intense itching and dyspareunia. Based on these findings, the nurse would suspect that the client has: A) Trichomoniasis B) Bacterial vaginosis C) Candidiasis D) Genital herpes simplex

C) Candidiasis

A woman gave birth to a newborn via vaginal delivery with the use of a vacuum extractor. The nurse would be alert for which of the following in the newborn? A) Asphyxia B) Clavicular fracture C) Caput succedaneum D) Central nervous system injury

C) Caput succedaneum

While performing a physical assessment of a newborn boy, the nurse notes diffuse edema of the soft tissues of his scalp that crosses suture lines. The nurse documents this finding as: A) Molding B) Microcephaly C) Caput succedaneum D) Cephalhematoma

C) Caput succedaneum

The nurse encourages a female client with human papillomavirus (HPV) to receive continued follow-up care because she is at risk for: A) Infertility B) Dyspareunia C) Cervical cancer D) Dysmenorrhea

C) Cervical cancer

During a vaginal exam, the nurse notes that the cervix has a bluish color. The nurse documents this finding as: A) Hegar's sign B) Goodell's sign C) Chadwick's sign D) Ortolani's sign

C) Chadwick's sign

On the first prenatal visit, examination of the woman's internal genitalia reveals a bluish coloration of the cervix and vaginal mucosa. The nurse records this finding as: A) Hegar's sign B) Goodell's sign C) Chadwick's sign D) Homans' sign

C) Chadwick's sign

A nurse is assisting with the delivery of a newborn. The fetal head has just emerged. Which of the following would be done next? A) Suctioning of the mouth and nose B) Clamping of the umbilical cord C) Checking for the cord around the neck D) Drying of the newborn

C) Checking for the cord around the neck

After teaching a class on sexually transmitted infections, the instructor determines that the teaching was successful when the class identifies which statement as true? A) STIs can affect anyone if exposed to the infectious organism. B) STIs have been addressed more on a global scale. C) Clients readily view the diagnosis of STI openly. D) Most individuals with STIs are over the age of 30.

C) Clients readily view the diagnosis of STI openly.

After discussing various methods of contraception with a client and her partner, the nurse determines that the teaching was successful when they identify which contraceptive method as providing protection against sexually transmitted infections (STIs)? A) Oral contraceptives B) Tubal ligation C) Condoms D) Intrauterine system

C) Condoms

Assessment of newborn reveals a large protruding tongue, slow reflexes, distended abdomen, poor feeding, hoarse cry, goiter and dry skin. Which of the following would the nurse suspect? A) Phenylketonuria B) Galactosemia C) Congenital hypothyroidism D) Maple syrup urine disease

C) Congenital hypothyroidism

A nurse is preparing a presentation for a group of young adult pregnant women about common infections and their effect on pregnancy. When describing the infections, which infection would the nurse include as the most common congenital and perinatal viral infection in the world? A) Rubella B) Hepatitis B C) Cytomegalovirus D) Parvovirus B19

C) Cytomegalovirus

A client is diagnosed with gestational hypertension and is receiving magnesium sulfate. Which finding would the nurse interpret as indicating a therapeutic level of medication? A) Urinary output of 20 mL per hour B) Respiratory rate of 10 breaths/minute C) Deep tendons reflexes 2+ D) Difficulty in arousing

C) Deep tendons reflexes 2+

The nurse frequently assesses the respiratory status of a preterm newborn based on the understanding that the newborn is at increased risk for respiratory distress syndrome because of which of the following? A) Inability to clear fluids B) Immature respiratory control center C) Deficiency of surfactant D) Smaller respiratory passages

C) Deficiency of surfactant

The nurse discusses various contraceptive methods with a client and her partner. Which method would the nurse explain as being available only with a prescription? A) Condom B) Spermicide C) Diaphragm D) Basal body temperature

C) Diaphragm

A breast biopsy indicates the presence of malignant cells, and the client is scheduled for a mastectomy. Which nursing diagnosis would the nurse most likely include in the client's preoperative plan of care as the priority? A) Risk for deficient fluid volume B) Activity intolerance C) Disturbed body image D) Impaired urinary elimination

C) Disturbed body image

A postpartum client has a fourth-degree perineal laceration. The nurse would expect which of the following medications to be ordered? A) Ferrous sulfate (Feosol) B) Methylergonovine (Methergine) C) Docusate (Colace) D) Bromocriptine (Parlodel)

C) Docusate (Colace)

A woman comes to the clinic reporting a nipple discharge. On examination, the area below the areola is red and slightly swollen, with tortuous tubular swelling. The nurse interprets these findings as suggestive of which of the following? A) Fibrocystic breast disorder B) Intraductal papilloma C) Duct ectasia D) Fibroadenoma

C) Duct ectasia

A nurse is assessing a pregnant woman who has come to the clinic. The woman reports that she feels some heaviness in her thighs since yesterday. The nurse suspects that the woman may be experiencing preterm labor based on which additional assessment findings? A) Dull low backache B) Malodorous vaginal discharge C) Dysuria D) Constipation

C) Dysuria

A nurse is preparing a class for a group of young adult women about emergency contraceptives (ECs). Which of the following would the nurse need to stress to the group. Select all that apply. A) ECs induce an abortion like reaction. B) ECs provide some protection against STIs C) ECs are birth control pills in higher, more frequent doses D) ECs are not to be used in place of regular birth control E) ECs provide little protection for future pregnancies.

C) ECs are birth control pills in higher, more frequent doses D) ECs are not to be used in place of regular birth control E) ECs provide little protection for future pregnancies.

A nurse is developing a plan of care for a woman who is at risk for thromboembolism. Which of the following would the nurse include as the most cost-effective method for prevention? A) Prophylactic heparin administration B) Compression stocking C) Early ambulation D) Warm compresses

C) Early ambulation

A newborn is suspected of developing persistent pulmonary hypertension. The nurse would expect to prepare the newborn for which of the following to confirm the suspicion? A) Chest x-ray B) Blood cultures C) Echocardiogram D) Stool for occult blood

C) Echocardiogram

After teaching a class on preventing pelvic inflammatory disease, the instructor determines that the teaching was successful when the class identifies which of the following as an effective method? A) Advising sexually active females to use hormonal contraception B) Encouraging vaginal douching on a weekly basis. C) Emphasizing the need for infected sexual partners to receive treatment D) Promoting routine treatment for asymptomatic females as risk

C) Emphasizing the need for infected sexual partners to receive treatment

When preparing a woman for an amniocentesis, the nurse would instruct her to do which of the following? A) Shower with an antiseptic scrub. B) Swallow the preprocedure sedative. C) Empty her bladder. D) Lie on her left side.

C) Empty her bladder.

A woman who has undergone a right modified-radical mastectomy returns from surgery. Which nursing intervention would be most appropriate at this time? A) Ask the client how she feels about having her breast removed. B) Attach a sign above her bed to have BP, IV lines, and lab work in her right arm. C) Encourage her to turn, cough, and deep breathe at frequent intervals. D) Position her right arm below heart level.

C) Encourage her to turn, cough, and deep breathe at frequent intervals.

The parents of a preterm newborn being cared for in the neonatal intensive care unit (NICU. are coming to visit for the first time. The newborn is receiving mechanical ventilation and intravenous fluids and medications and is being monitored electronically by various devices. Which action by the nurse would be most appropriate? A) Suggest that the parents stay for just a few minutes to reduce their anxiety. B) Reassure them that their newborn is progressing well. C) Encourage the parents to touch their preterm newborn. D) Discuss the care they will be giving the newborn upon discharge.

C) Encourage the parents to touch their preterm newborn.

The nurse is assessing the skin of a newborn and notes a rash on the newborn's face, and chest. The rash consists of small papules and is scattered with no pattern. The nurse interprets this finding as which of the following? A) Harlequin sign B) Nevus flammeus C) Erythema toxicum D) Port wine stain

C) Erythema toxicum

When planning the care of a woman in the active phase of labor, the nurse would anticipate assessing the fetal heart rate at which interval? A) Every 2 to 4 hours B) Every 45 to 60 minutes C) Every 15 to 30 minutes D) Every 10 to 15 minutes

C) Every 15 to 30 minutes

A nurse is providing teaching to a new mother about her newborn's nutritional needs. Which of the following would the nurse be most likely to include in the teaching? (Select all that apply.) A) Supplementing with iron if the woman is breast-feeding B) Providing supplemental water intake with feedings C) Feeding the newborn every 2 to 4 hours during the day D) Burping the newborns frequently throughout each feeding E) Using feeding time for promoting closeness

C) Feeding the newborn every 2 to 4 hours during the day D) Burping the newborns frequently throughout each feeding E) Using feeding time for promoting closeness

After teaching a group of students about genital fistulas, the instructor determines that the teaching was successful when the students identify which of the following as a major cause? A) Radiation therapy B) Congenital anomaly C) Female genital cutting D) Bartholin's gland abscess

C) Female genital cutting

After teaching a group of nursing students about the possible causes of spontaneous abortion, the instructor determines that the teaching was successful when the students identify which of the following as the most common cause of first trimester abortions? A) Maternal disease B) Cervical insufficiency C) Fetal genetic abnormalities D) Uterine fibroids

C) Fetal genetic abnormalities

A group of students are reviewing information about benign and malignant breast masses. The students demonstrate understanding when they identify which of the following as indicating a benign breast mass. (Select all the apply.) A) Painless B) Unilateral location C) Firm consistency D) Absence of dimpling E) Fixed to chest wall

C) Firm consistency D) Absence of dimpling

Prenatal testing is used to assess for genetic risks and to identify genetic disorders. In explaining to a couple about an elevated alpha-fetoprotein screening test result, the nurse would discuss the need for: A) Special care needed for a Down syndrome infant B) A more specific determination of the acid-base status C) Further, more definitive evaluations to conclude anything D) Immediate termination of the pregnancy based on results

C) Further, more definitive evaluations to conclude anything

A group of nursing students are reviewing information about vaccines used to prevent STIs. The students would expect to find information about which of the following? A) HIV B) HSV C) HPV D) HAV E) HBV

C) HPV D) HAV E) HBV

Which of the following would the nurse expect to assess in a newborn who develops sepsis? A) Increased urinary output B) Interest in feeding C) Hypothermia D) Wakefulness

C) Hypothermia

Evaluation of a woman with breast cancer reveals that her mass is approximately 1.25 inches in diameter. Three adjacent lymph nodes are positive. The nurse interprets this as indicating that the woman has which stage of breast cancer? A) 0 B) I C) II D) III

C) II

A pregnant woman has a rubella titer drawn on her first prenatal visit. The nurse explains that this test measures which of the following? A) Platelet level B) Rh status C) Immunity to German measles D) Red blood cell count

C) Immunity to German measles

The nurse develops a teaching plan for a postpartum client and includes teaching about how to perform Kegel exercises. The nurse includes this information for which reason? A) Reduce lochia B) Promote uterine involution C) Improve pelvic floor tone D) Alleviate perineal pain

C) Improve pelvic floor tone

The nurse is developing a plan of care for a client who is receiving highly active antiretroviral therapy (HAART) for treatment of HIV. The goal of this therapy is to: A) Promote the progression of disease B) Intervene in late-stage AIDS C) Improve survival rates D) Conduct additional drug research

C) Improve survival rates

Which of the following changes in the musculoskeletal system would the nurse mention when teaching a group of pregnant women about the physiologic changes of pregnancy? A) Ligament tightening B) Decreased swayback C) Increased lordosis D) Joint contraction

C) Increased lordosis

A group of nursing students are reviewing the changes in the newborn's lungs that must occur to maintain respiratory function. The students demonstrate understanding of this information when they identify which of the following as the first event? A) Expansion of the lungs B) Increased pulmonary blood flow C) Initiation of respiratory movement D) Redistribution of cardiac output

C) Initiation of respiratory movement

A nurse is teaching the mother of a newborn diagnosed with galactosemia about dietary restrictions. The nurse determines that the mother has understood the teaching when she identifies which of the following as needing to be restricted? A) Phenylalanine B) Protein C) Lactose D) Iodine

C) Lactose

A client comes to the clinic with abdominal pain. Based on her history the nurse suspects endometriosis. The nurse expects to prepare the client for which of the following to confirm this suspicion? A) Pelvic examination B) Transvaginal ultrasound C) Laparoscopy D) Hysterosalpingogram

C) Laparoscopy

A nurse is documenting fetal lie of a woman in labor. Which term would the nurse most likely use? A) Flexion B) Extension C) Longitudinal D) Cephalic

C) Longitudinal

The health care provider orders PGE2 for a woman to help evacuate the uterus following a spontaneous abortion. Which of the following would be most important for the nurse to do? A) Use clean technique to administer the drug. B) Keep the gel cool until ready to use. C) Maintain the client for ½ hour after administration. D) Administer intramuscularly into the deltoid area.

C) Maintain the client for ½ hour after administration.

A group of students are reviewing information about the effects of substances on the newborn. The students demonstrate understanding of the information when they identify which drug as not being associated with teratogenic effects on the fetus? A) Alcohol B) Nicotine C) Marijuana D) Cocaine

C) Marijuana

When reviewing the medical record of a client diagnosed with endometriosis, which of the following would the nurse identify as a risk factor for this woman? A) Low fat in the diet B) Age of 14 years for menarche C) Menstrual cycles of 24 days D) Short menstrual flow

C) Menstrual cycles of 24 days

When palpating the fundus during a contraction, the nurse notes that it feels like a chin. The nurse interprets this finding as indicating which type of contraction? A) Intense B) Strong C) Moderate D) Mild

C) Moderate

A woman in labor who received an opioid for pain relief develops respiratory depression. The nurse would expect which agent to be administered? A) Butorphanol B) Fentanyl C) Naloxone D) Promethazine

C) Naloxone

The nurse completes the initial assessment of a newborn. Which finding would lead the nurse to suspect that the newborn is experiencing difficulty with oxygenation? A) Respiratory rate of 54 breaths/minute B) Abdominal breathing C) Nasal flaring D) Acrocyanosis

C) Nasal flaring

A client with hyperemesis gravidarum is admitted to the facility after being cared for at home without success. Which of the following would the nurse expect to include in the client's plan of care? A) Clear liquid diet B) Total parenteral nutrition C) Nothing by mouth D) Administration of labetalol

C) Nothing by mouth

When assessing fetal heart rate, the nurse finds a heart rate of 175 bpm, accompanied by a decrease in variability and late decelerations. Which of the following would the nurse do next? A) Have the woman change her position. B) Administer oxygen. C) Notify the health care provider. D) Continue to monitor the pattern every 15 minutes.

C) Notify the health care provider.

A woman in labor is experiencing hypotonic uterine dysfunction. Assessment reveals no fetopelvic disproportion. Which group of medications would the nurse expect to administer? A) Sedatives B) Tocolytics C) Oxytocins D) Corticosteroids

C) Oxytocins

A group of nursing students are reviewing the literature in preparation for a class presentation on newborn pain prevention and management. Which of the following would the students be most likely to find about this topic? A) Newborn pain is frequently recognized and treated B) Newborns rarely experience pain with procedures C) Pain is frequently mistaken for irritability or agitation D) Newborns may be less sensitive to pain than adult.

C) Pain is frequently mistaken for irritability or agitation

A woman suspecting she is pregnant asks the nurse about which signs would confirm her pregnancy. The nurse would explain that which of the following would confirm the pregnancy? A) Absence of menstrual period B) Abdominal enlargement C) Palpable fetal movement D) Morning sickness

C) Palpable fetal movement

When teaching a woman how to perform Kegel exercises, the nurse explains that these exercises are designed to strengthen which muscles? A) Gluteus B) Lower abdominal C) Pelvic floor D) Diaphragmatic

C) Pelvic floor

A nurse is describing the criteria needed for the diagnosis of premenstrual dysphoric disorder (PMDD). Which of the following would the nurse include as a mandatory requirement for the diagnosis? A) Appetite changes B) Sleep difficulties C) Persistent anger D) Chronic fatigue

C) Persistent anger

A nurse is preparing an inservice education program for a group of nurses about dystocia involving problems with the passenger. Which of the following would the nurse most likely include as the most common problem? A) Macrosomia B) Breech presentation C) Persistent occiput posterior position D) Multifetal pregnancy

C) Persistent occiput posterior position

A postpartum woman is ordered to receive oxytocin to stimulate the uterus to contract. Which of the following would be most important for the nurse to do? A) Administer the drug as an IV bolus injection. B) Give as a vaginal or rectal suppository. C) Piggyback the IV infusion into a primary line. D) Withhold the drug if the woman is hypertensive.

C) Piggyback the IV infusion into a primary line

A postpartum woman is having difficulty voiding for the first time after giving birth. Which of the following would be least effective in helping to stimulate voiding? A) Pouring warm water over her perineal area B) Having her hear the sound of water running nearby C) Placing her hand in a basin of cool water D) Standing her in the shower with the warm water on

C) Placing her hand in a basin of cool water

Which of the following data on a client's health history would the nurse identify as contributing to the client's risk for an ectopic pregnancy? A) Use of oral contraceptives for 5 years B) Ovarian cyst 2 years ago C) Recurrent pelvic infections D) Heavy, irregular menses

C) Recurrent pelvic infections

When explaining how a newborn adapts to extrauterine life, the nurse would describe which body systems as undergoing the most rapid changes? A) Gastrointestinal and hepatic B) Urinary and hematologic C) Respiratory and cardiovascular D) Neurological and integumentary

C) Respiratory and cardiovascular

After teaching a group of nursing students about risk factors associated with dystocia, the instructor determines that the teaching was successful when the students identify which of the following as increasing the risk? (Select all that apply.) A) Pudendal block anesthetic use B) Multiparity C) Short maternal stature D) Maternal age over 35 E) Breech fetal presentation

C) Short maternal stature D) Maternal age over 35 E) Breech fetal presentation

The nurse is teaching a group of students about the similarities and differences between newborn skin and adult skin. Which statement by the group indicates that additional teaching is needed? A) The newborn's skin and that of an adult are similar in thickness. B) The lipid composition of the skin of a newborn and adult is about the same. C) Skin development in the newborn is complete at birth. D) The newborn has more fibrils connecting the dermis and epidermis.

C) Skin development in the newborn is complete at birth.

While observing the interaction between a newborn and his mother, the nurse notes the newborn nestling into the arms of his mother. The nurse identifies this behavior as which of the following? A) Habituation B) Self-quieting ability C) Social behaviors D) Orientation

C) Social behaviors

A nurse is providing care to a woman during the third stage of labor. Which of the following would alert the nurse that the placenta is separating? (Select all that apply.) A) Boggy, soft uterus B) Uterus becoming discoid shaped C) Sudden gush of dark blood from the vagina D) Shortening of the umbilical cord

C) Sudden gush of dark blood from the vagina

A woman hospitalized with severe preeclampsia is being treated with hydralazine to control blood pressure. Which of the following would the lead the nurse to suspect that the client is having an adverse effect associated with this drug? A) Gastrointestinal bleeding B) Blurred vision C) Tachycardia D) Sweating

C) Tachycardia

A nurse is completing a postpartum assessment. Which finding would alert the nurse to a potential problem? A) Lochia rubra with a fleshy odor B) Respiratory rate of 16 breaths per minute C) Temperature of 101° F D) Pain rating of 2 on a scale from 0 to 10

C) Temperature of 101° F

A nursing student is preparing a presentation for the class on clubfoot. The student determines that the presentation was successful when the class states which of the following? A) Clubfoot is a common genetic disorder. B) The condition affects girls more often than boys. C) The exact cause of clubfoot is not known. D) The intrinsic form can be manually reduced.

C) The exact cause of clubfoot is not known.

The nurse is assessing a pregnant woman in the second trimester. Which of the following tasks would indicate to the nurse that the client is incorporating the maternal role into her personality? A) The woman demonstrates concern for herself and her fetus as a unit. B) The client identifies what she must give up to assume her new role. C) The woman acknowledges the fetus as a separate entity within her. D) The client demonstrates unconditional acceptance without rejection.

C) The woman acknowledges the fetus as a separate entity within her.

When assessing a postterm newborn, which of the following would the nurse correlate with this gestational age variation? A) Moist, supple, plum skin appearance B) Abundant lanugo and vernix C) Thin umbilical cord D) Absence of sole creases

C) Thin umbilical cord

Review of a primiparous woman's labor and birth record reveals a prolonged second stage of labor and extended time in the stirrups. Based on an interpretation of these findings, the nurse would be especially alert for which of the following? A) Retained placental fragments B) Hypertension C) Thrombophlebitis D) Uterine subinvolution

C) Thrombophlebitis

When reviewing the medical record of a postpartum client, the nurse notes that the client has experienced a third-degree laceration. The nurse understands that the laceration extends to which of the following? A) Superficial structures above the muscle B) Through the perineal muscles C) Through the anal sphincter muscle D) Through the anterior rectal wall

C) Through the anal sphincter muscle

The nurse encourages the mother of a healthy newborn to put the newborn to the breast immediately after birth for which reason? A) To aid in maturing the newborn's sucking reflex B) To encourage the development of maternal antibodies C) To facilitate maternal-infant bonding D) To enhance the clearing of the newborn's respiratory passages

C) To facilitate maternal-infant bonding

A client has not received any medication during her labor. She is having frequent contractions every 1 to 2 minutes and has become irritable with her coach and no longer will allow the nurse to palpate her fundus during contractions. Her cervix is 8 cm dilated and 90% effaced. The nurse interprets these findings as indicating: A) Latent phase of the first stage of labor B) Active phase of the first stage of labor C) Transition phase of the first stage of labor D) Pelvic phase of the second stage of labor

C) Transition phase of the first stage of labor

While talking with a pregnant woman who has undergone genetic testing, the woman informs the nurse that her baby will be born with Down syndrome. The nurse understands that Down syndrome is an example of: A) Multifactorial inheritance B) X-linked recessive inheritance C) Trisomy numeric abnormality D) Chromosomal deletion

C) Trisomy numeric abnormality

A woman is in the first stage of labor. The nurse would encourage her to assume which position to facilitate the progress of labor? A) Supine B) Lithotomy C) Upright D) Knee-chest

C) Upright

Which of the following factors in a client's history would alert the nurse to an increased risk for postpartum hemorrhage? A) Multiparity, age of mother, operative delivery B) Size of placenta, small baby, operative delivery C) Uterine atony, placenta previa, operative procedures D) Prematurity, infection, length of labor

C) Uterine atony, placenta previa, operative procedures

A woman with a history of crack cocaine abuse is admitted to the labor and birth area. While caring for the client, the nurse notes a sudden onset of fetal bradycardia. Inspection of the abdomen reveals an irregular wall contour. The client also complains of acute abdominal pain that is continuous. Which of the following would the nurse suspect? A) Amniotic fluid embolism B) Shoulder dystocia C) Uterine rupture D) Umbilical cord prolapse

C) Uterine rupture

Which of the following would be most appropriate when massaging a woman's fundus? A) Place the hands on the sides of the abdomen to grasp the uterus. B) Use an up-and-down motion to massage the uterus. C) Wait until the uterus is firm to express clots. D) Continue massaging the uterus for at least 5 minutes.

C) Wait until the uterus is firm to express clots.

When the nurse is assessing a postpartum client approximately 6 hours after delivery, which finding would warrant further investigation? A) Deep red, fleshy-smelling lochia B) Voiding of 350 cc C) Heart rate of 120 beats/minute D) Profuse sweating

C. Heart rate of 120 beats/minute

When caring for a mother who has had a cesarean birth, the nurse would expect the client's lochia to be: a.Greater than after a vaginal delivery b. About the same as after a vaginal delivery c. Less than after a vaginal delivery d. Saturated with clots and mucus

C. less than after a vaginal delivery

When developing a teaching plan for a couple considering contraception options, which of the following statements would the nurse include? A) "You should select one that is considered to be 100% effective." B) "The best one is the one that is the least expensive and most convenient." C) "A good contraceptive doesn't require a physician's prescription." D) "The best contraceptive is one that you will use correctly and consistently."

D) "The best contraceptive is one that you will use correctly and consistently."

A postmenopausal woman with uterine prolapse is being fitted with a pessary. The nurse would be most alert for which side effect? A) Increased vaginal discharge B) Urinary tract infection C) Vaginitis D) Vaginal ulceration

D) Vaginal ulceration

A client who is HIV-positive is in her second trimester and remains asymptomatic. She voices concern about her newborn's risk for the infection. Which of the following statements by the nurse would be most appropriate? A) "You'll probably have a cesarean birth to prevent exposing your newborn." B) "Antibodies cross the placenta and provide immunity to the newborn." C) "Wait until after the infant is born and then something can be done." D) "Antiretroviral medications are available to help reduce the risk of transmission."

D) "Antiretroviral medications are available to help reduce the risk of transmission."

A nurse is assisting the anxious parents of a preterm newborn to cope with the situation. Which statement by the nurse would be least appropriate? A) "I'll be here to help you all along the way." B) "What has helped you to deal with stressful situations in the past?" C) "Let me tell you about what you will see when you visit your baby." D) "Forget about what's happened in the past and focus on the now."

D) "Forget about what's happened in the past and focus on the now."

When assessing a pregnant woman in her last trimester, which question would be most appropriate to use to gather information about weight gain and fluid retention? A) "What's your usual dietary intake for a typical day?" B) "What size maternity clothes are you wearing now?" C) "How puffy does your face look by the end of a day?" D) "How swollen do your ankles appear before you go to bed?

D) "How swollen do your ankles appear before you go to bed?

A nurse is teaching a pregnant woman at risk for preterm labor about what to do if she experiences signs and symptoms. The nurse determines that the teaching was successful when the woman states that if she experiences any symptoms, she will do which of the following? A) "I'll sit down to rest for 30 minutes." B) "I'll try to move my bowels." C) "I'll lie down with my legs raised." D) "I'll drink several glasses of water."

D) "I'll drink several glasses of water."

A nurse is conducting a class on breast cancer prevention. Which statement would the nurse most likely include in the discussion? A) "Most often a lump is felt before it is seen." B) "Early breast cancer usually has some symptoms." C) "If the mass is not painful, it is usually benign." D) "If lump is palpable, it has been there for some time."

D) "If lump is palpable, it has been there for some time."

A pregnant woman asks the nurse, "I'm a big coffee drinker. Will the caffeine in my coffee hurt my baby?" Which response by the nurse would be most appropriate? A) "The caffeine in coffee has been linked to birth defects." B) "Caffeine has been shown to cause growth restriction in the fetus." C) "Caffeine is a stimulant and needs to be avoided completely." D) "If you keep your intake to less than 300 mg/day, you should be okay."

D) "If you keep your intake to less than 300 mg/day, you should be okay."

Which instructions would the nurse include when teaching a woman with pediculosis pubis? A) "Take the antibiotic until you feel better." B) "Wash your bed linens in bleach and cold water." C) "Your partner doesn't need treatment at this time." D) "Remove the nits with a fine-toothed comb."

D) "Remove the nits with a fine-toothed comb."

A client who is breast-feeding her newborn tells the nurse, "I notice that when I feed him, I feel fairly strong contraction-like pain. Labor is over. Why am I having contractions now?" Which response by the nurse would be most appropriate? A) "Your uterus is still shrinking in size; that's why you're feeling this pain." B) "Let me check your vaginal discharge just to make sure everything is fine." C) "Your body is responding to the events of labor, just like after a tough workout." D) "The baby's sucking releases a hormone that causes the uterus to contract."

D) "The baby's sucking releases a hormone that causes the uterus to contract."

A nurse is caring for several women in labor. The nurse determines that which woman is in the transition phase of labor? A) Contractions every 5 minutes, cervical dilation 3 cm B) Contractions every 3 minutes, cervical dilation 5 cm C) Contractions every 2½ minutes, cervical dilation 7 cm D) Contractions every 1 minute, cervical dilation 9 cm

D) Contractions every 1 minute, cervical dilation 9 cm

A woman in labor has chosen to use hydrotherapy as a method of pain relief. Which statement by the woman would lead the nurse to suspect that the woman needs additional teaching? A) "The warmth and buoyancy of the water has a nice relaxing effect." B) "I can stay in the bath for as long as I feel comfortable." C) "My cervix should be dilated more than 5 cm before I try using this method." D) "The temperature of the water should be at least 105° F."

D) "The temperature of the water should be at least 105° F."

A fetus is assessed at 2 cm above the ischial spines. The nurse would document fetal station as: A) +4 B) +2 C) 0 D) -2

D) -2

A woman is being evaluated for pelvic organ prolapse. A postvoid residual urine specimen is obtained via a catheter. Which residual volume finding would lead the nurse to suspect the need for further testing? A) 50 mL B) 75 mL C) 100 mL D) 120 mL

D) 120 mL

The nurse is teaching a pregnant woman about recommended weight gain. The woman has a prepregnancy body mass index of 26. The nurse determines that the teaching was successful when the woman states that she should gain no more than which amount during pregnancy? A) 35 to 40 pounds B) 25 to 35 pounds C) 28 to 40 pounds D) 15 to 25 pounds

D) 15 to 25 pounds

The nurse reviews the CD4 cell count of a client who is HIV-positive. A result less than which of the following would indicate to the nurse that the client has AIDS? A) 1,000 cells/mm3 B) 700 cells/mm3 C) 450 cells/mm3 D) 200 cells/mm3

D) 200 cells/mm3

When assessing a pregnant woman with heart disease throughout the antepartal period, the nurse would be especially alert for signs and symptoms of cardiac decompensation at which time? A) 16 to 20 weeks' gestation B) 20 to 24 weeks' gestation C) 24 to 28 weeks' gestation D) 28 to 32 weeks' gestation

D) 28 to 32 weeks' gestation

A nurse is providing care to several pregnant women at the clinic. The nurse would screen for group B streptococcus infection in a client at: A) 16 weeks' gestation B) 28 week' gestation C) 32 weeks' gestation D) 36 weeks' gestation

D) 36 weeks' gestation

A couple comes to the clinic for preconception counseling and care. As part of the visit, the nurse teaches the couple about fertilization and initial development, stating that the zygote formed by the union of the ovum and sperm consists of how many chromosomes? A) 22 B) 23 C) 44 D) 46

D) 46

The nurse is assessing the respirations of several newborns. The nurse would notify the health care provider for the newborn with which respiratory rate at rest? A) 38 breaths per minute B) 46 breaths per minute C) 54 breaths per minute D) 68 breaths per minute

D) 68 breaths per minute

The nurse institutes measure to maintain thermoregulation based on the understanding that newborns have limited ability to regulate body temperature because they: A) Have a smaller body surface compared to body mass B) Lose more body heat when they sweat than adults C) Have an abundant amount of subcutaneous fat all over D) Are unable to shiver effectively to increase heat production

D) Are unable to shiver effectively to increase heat production

While making rounds in the nursery, the nurse sees a 6-hour-old baby girl gagging and turning bluish. Which of the following would the nurse do first? A) Alert the physician stat and turn the newborn to her right side. B) Administer oxygen via facial mask by positive pressure. C) Lower the newborn's head to stimulate crying. D) Aspirate the oral and nasal pharynx with a bulb syringe.

D) Aspirate the oral and nasal pharynx with a bulb syringe.

It is determined that a client's blood Rh is negative and her partner's is positive. To help prevent Rh isoimmunization, the nurse anticipates that the client will receive RhoGAM at which time? A) At 34 weeks' gestation and immediately before discharge B) 24 hours before delivery and 24 hours after delivery C) In the first trimester and within 2 hours of delivery D) At 28 weeks' gestation and again within 72 hours after delivery

D) At 28 weeks' gestation and again within 72 hours after delivery

Which of the following would alert the nurse to suspect that a newborn has developed NEC? A) Irritability B) Sunken abdomen C) Clay-colored stools D) Bilious vomiting

D) Bilious vomiting

A client states, "I think my waters broke! I felt this gush of fluid between my legs." The nurse tests the fluid with Nitrazine paper and confirms membrane rupture if the paper turns: A) Yellow B) Olive green C) Pink D) Blue

D) Blue

As part of an inservice program, a nurse is describing a transient, self-limiting mood disorder that affects mothers after childbirth. The nurse correctly identifies this as postpartum: A) Depression B) Psychosis C) Bipolar disorder D) Blues

D) Blues

Assessment of a pregnant woman and her fetus reveals tachycardia and hypertension. There is also evidence suggesting vasoconstriction. The nurse would question the woman about use of which substance? A) Marijuana B) Alcohol C) Heroin D) Cocaine

D) Cocaine

The nurse places a newborn with jaundice under the phototherapy lights in the nursery to achieve which goal? A) Prevent cold stress B) Increase surfactant levels in the lungs C) Promote respiratory stability D) Decrease the serum bilirubin level

D) Decrease the serum bilirubin level

The nurse is reviewing the laboratory test results of a pregnant client. Which one of the following findings would alert the nurse to the development of HELLP syndrome? A) Hyperglycemia B) Elevated platelet count C) Leukocytosis D) Elevated liver enzymes

D) Elevated liver enzymes

A group of students are preparing a class presentation about polyps. Which of the following would the students most likely include in the presentation? A) Polyps are rarely the result of an infection. B) Endocervical polyps commonly appear after menarche. C) Cervical polyps are more common than endocervical polyps. D) Endocervical polyps are most common in women in their 50s.

D) Endocervical polyps are most common in women in their 50s.

Assessment of a woman in labor who is experiencing hypertonic uterine dysfunction would reveal contractions that are: A) Well coordinated B) Poor in quality C) Rapidly occurring D) Erratic

D) Erratic

A neonate born to a mother who was abusing heroin is exhibiting signs and symptoms of withdrawal. Which of the following would the nurse assess? (Select all that apply.) A) Low whimpering cry B) Hypertonicity C) Lethargy D) Excessive sneezing E) Overly vigorous sucking F) Tremors

D) Excessive sneezing

After teaching a group of students about pelvic organ prolapse, the instructor determines that the teaching was successful when the group identifies leiomyomas as which of the following? A) Cysts B) Pelvic organ prolapse C) Fistula D) Fibroid

D) Fibroid

Which assessment finding would lead the nurse to suspect infection as the cause of a client's PROM? A) Yellow-green fluid B) Blue color on Nitrazine testing C) Ferning D) Foul odor

D) Foul odor

When assessing a woman at follow-up prenatal visits, the nurse would anticipate which of the following to be performed? A) Hemoglobin and hematocrit B) Urine for culture C) Fetal ultrasound D) Fundal height measurement

D) Fundal height measurement

Which of the following findings on a prenatal visit at 10 weeks might lead the nurse to suspect a hydatidiform mole? A) Complaint of frequent mild nausea B) Blood pressure of 120/84 mm Hg C) History of bright red spotting 6 weeks ago D) Fundal height measurement of 18 cm

D) Fundal height measurement of 18 cm

Which of the following findings would the nurse interpret as suggesting a diagnosis of gestational trophoblastic disease? A) Elevated hCG levels, enlarged abdomen, quickening B) Vaginal bleeding, absence of FHR, decreased hPL levels C) Visible fetal skeleton on ultrasound, absence of quickening, enlarged abdomen D) Gestational hypertension, hyperemesis gravidarum, absence of FHR

D) Gestational hypertension, hyperemesis gravidarum, absence of FHR

When teaching a pregnant client about the physiologic changes of pregnancy, the nurse reviews the effect of pregnancy on glucose metabolism. Which of the following would the nurse include as the underlying reason for the effect? A) Pancreatic function is affected by pregnancy. B) Glucose is utilized more rapidly during a pregnancy. C) The pregnant woman increases her dietary intake. D) Glucose moves through the placenta to assist the fetus.

D) Glucose moves through the placenta to assist the fetus.

A woman gives birth to a healthy newborn. As part of the newborn's care, the nurse instills erythromycin ophthalmic ointment as a preventive measure related to which STI? A) Genital herpes B) Hepatitis B C) Syphilis D) Gonorrhea

D) Gonorrhea

The nurse is teaching a group of students about the differences between a full-term newborn and a preterm newborn. The nurse determines that the teaching is effective when the students state that the preterm newborn has: A) Fewer visible blood vessels through the skin B) More subcutaneous fat in the neck and abdomen C) Well-developed flexor muscles in the extremities D) Greater surface area in proportion to weight

D) Greater surface area in proportion to weight

A woman who is 2 weeks postpartum calls the clinic and says, "My left breast hurts." After further assessment on the phone, the nurse suspects the woman has mastitis. In addition to pain, the nurse would assess for which of the following? A) An inverted nipple on the affected breast B) No breast milk in the affected breast C) An ecchymotic area on the affected breast D) Hardening of an area in the affected breast

D) Hardening of an area in the affected breast

A pregnant client in her second trimester has a hemoglobin level of 11 g/dL. The nurse interprets this as indicating which of the following? A) Iron-deficiency anemia B) A multiple gestation pregnancy C) Greater-than-expected weight gain D) Hemodilution of pregnancy

D) Hemodilution of pregnancy

After teaching a pregnant woman about the hormones produced by the placenta, the nurse determines that the teaching was successful when the woman identifies which hormone produced as being the basis for pregnancy tests? A) Human placental lactogen (hPL) B) Estrogen (estriol) C) Progesterone (progestin) D) Human chorionic gonadotropin (hCG)

D) Human chorionic gonadotropin (hCG)

A nurse is assessing a pregnant woman with gestational hypertension. Which of the following would lead the nurse to suspect that the client has developed severe preeclampsia? A) Urine protein 300 mg/24 hours B) Blood pressure 150/96 mm Hg C) Mild facial edema D) Hyperreflexia

D) Hyperreflexia

A nurse teaches a postpartum woman about her risk for thromboembolism. Which of the following would the nurse be least likely to include as a factor increasing her risk? A) Increased clotting factors B) Vessel damage C) Immobility D) Increased red blood cell production

D) Increased red blood cell production

When performing a clinical breast examination, which would the nurse do first? A) Palpate the axillary area. B) Compress the nipple for a discharge. C) Palpate the breasts. D) Inspect the breasts.

D) Inspect the breasts.

When discussing contraceptive options, which method would the nurse recommend as being the most reliable? A) Coitus interruptus B) Lactational amenorrheal method (LAM) C) Natural family planning D) Intrauterine system

D) Intrauterine system

After spontaneous rupture of membranes, the nurse notices a prolapsed cord. The nurse immediately places the woman in which position? A) Supine B) Side-lying C) Sitting D) Knee-chest

D) Knee-chest

A group of nursing students are reviewing the various medications used for pain relief during labor. The students demonstrate understanding of the information when they identify which agent as the most commonly used opioid? A) Butorphanol B) Nalbuphine C) Fentanyl D) Meperidine

D) Meperidine

A nurse is counseling a pregnant woman with rheumatoid arthritis about medications that can be used during pregnancy. Which drug would the nurse emphasize as being contraindicated at this time? A) Hydroxychloroquine B) Nonsteroidal anti-inflammatory drug C) Glucocorticoid D) Methotrexate

D) Methotrexate

A nurse is assessing a female client and suspects that the client may have endometrial polyps based on which of the following? A) Bleeding after intercourse B) Vaginal discharge C) Bleeding between menses D) Metrorrhagia

D) Metrorrhagia

A newborn has been diagnosed with a Group B streptococcal infection shortly after birth. The nurse understands that the newborn most likely acquired this infection from which of the following? A) Improper handwashing B) Contaminated formula C) Nonsterile catheter insertion D) Mother's birth canal

D) Mother's birth canal

A woman with breast cancer is undergoing chemotherapy. Which of the following side effects would the nurse interpret as being most serious? A) Vomiting B) Hair loss C) Fatigue D) Myelosuppression

D) Myelosuppression

A client who has come to the clinic is diagnosed with endometriosis. Which of the following would the nurse expect the physician to prescribe as a first-line treatment? A) Progestins B) Antiestrogens C) Gonadotropin-releasing hormone analogues D) NSAIDs

D) NSAIDs

When assessing a client with suspected breast cancer, which of the following would the nurse expect to find? A) Painful lump B) Absence of dimpling C) Regularly shaped mass D) Nipple retraction

D) Nipple retraction

Which medication would the nurse question if ordered to control a pregnant woman's asthma? A) Budesonide B) Albuterol C) Salmeterol D) Oral prednisone

D) Oral prednisone

A nurse is assessing a newborn and observes the newborn moving his head and eyes toward a loud sound. The nurse interprets this as which of the following? A) Habituation B) Motor maturity C) Social behavior D) Orientation

D) Orientation

A group of students are reviewing the process of breast milk production. The students demonstrate understanding when they identify which hormone as responsible for milk let-down? A) Prolactin B) Estrogen C) Progesterone D) Oxytocin

D) Oxytocin

Which of the following measures would the nurse include in the teaching plan for a woman to reduce the risk of osteoporosis after menopause? A) Taking vitamin supplements B) Eating high-fiber, high-calorie foods C) Restricting fluid to 1,000 mL daily D) Participating in regular daily exercise

D) Participating in regular daily exercise

A nurse is working with a woman who has been diagnosed with severe fibrocystic breast disease. When describing the medications that can be used as treatment, which of the following would the nurse be least likely to include? A) Tamoxifen B) Bromocriptine C) Danazol D) Penicillin

D) Penicillin

The nurse administers vitamin K intramuscularly to the newborn based on which of the following rationales? A) Stop Rh sensitization B) Increase erythopoiesis C) Enhance bilirubin breakdown D) Promote blood clotting

D) Promote blood clotting

While talking with a woman in her third trimester, which behavior indicates to the nurse that the woman is learning to give of oneself? A) Showing concern for self and fetus as a unit B) Unconditionally accepting the pregnancy without rejection C) Longing to hold infant D) Questioning ability to become a good mother

D) Questioning ability to become a good mother

During a nonstress test, when monitoring the fetal heart rate, the nurse notes that when the expectant mother reports fetal movement, the heart rate increases 15 beats or more above the baseline. The nurse interprets this as: A) Variable decelerations B) Fetal tachycardia C) A nonreactive pattern D) Reactive pattern

D) Reactive pattern

A new mother reports that her newborn often spits up after feeding. Assessment reveals regurgitation. The nurse responds integrating understanding that this most likely is due to which of the following? A) Placing the newborn prone after feeding B) Limited ability of digestive enzymes C) Underdeveloped pyloric sphincter D) Relaxed cardiac sphincter

D) Relaxed cardiac sphincter

A nurse is assessing a woman in labor. Which finding would the nurse identify as a cause for concern during a contraction? A) Heart rate increase from 76 bpm to 90 bpm B) Blood pressure rise from 110/60 mm Hg to 120/74 C) White blood cell count of 12,000 cells/mm3 D) Respiratory rate of 10 breaths /minute

D) Respiratory rate of 10 breaths /minute

When assessing a newborn's reflexes, the nurse strokes the newborn's cheek and the newborn turns toward the side that was stroked and begins sucking. The nurse documents which reflex as being positive? A) Palmar grasp reflex B) Tonic neck reflex C) Moro reflex D) Rooting reflex

D) Rooting reflex

A nurse is assessing a postpartum woman. Which finding would cause the nurse to be most concerned? A) Leg pain on ambulation with mild ankle edema B) Calf pain with dorsiflexion of the foot. C) Perineal pain with swelling along the episiotomy D) Sharp stabbing chest pain with shortness of breath

D) Sharp stabbing chest pain with shortness of breath

Which finding would indicate to the nurse that a woman's cervix is ripe in preparation for labor induction? A) Posterior position B) Firm C) Closed D) Shortened

D) Shortened

A woman with diabetes is considering becoming pregnant. She asks the nurse whether she will be able to take oral hypoglycemics when she is pregnant. The nurse's response is based on the understanding that oral hypoglycemics: A) Can be used as long as they control serum glucose levels B) Can be taken until the degeneration of the placenta occurs C) Are usually suggested primarily for women who develop gestational diabetes D) Show promising results but more studies are needed to confirm their effectiveness

D) Show promising results but more studies are needed to confirm their effectiveness

A postpartum client comes to the clinic for her 6-week postpartum checkup. When assessing the client's cervix, the nurse would expect the external cervical os to appear: A) Shapeless B) Circular C) Triangular D) Slit-like

D) Slit-like

After teaching a local woman's group about incontinence, the nurse determines that the teaching was successful when the group identifies which of the following as characteristic of stress incontinence? A) Feeling a strong need to void B) Passing a large amount of urine C) Most common in women after childbirth D) Sneezing may be an initiating stimulus

D) Sneezing may be an initiating stimulus

Which of the following would alert the nurse to the possibility of respiratory distress in a newborn? A) Symmetrical chest movements B) Periodic breathing C) Respirations of 40 breaths/minute D) Sternal retractions

D) Sternal retractions

A woman is admitted for repair of cystocele and rectocele. She has nine living children. In taking her health history, which of the following would the nurse expect to find? A) Sporadic vaginal bleeding accompanied by chronic pelvic pain B) Heavy leukorrhea with vulvar pruritus C) Menstrual irregularities and hirsutism on the chin D) Stress incontinence with feeling of low abdominal pressure

D) Stress incontinence with feeling of low abdominal pressure

When performing newborn resuscitation, which action would the nurse do first? A) Intubate with an appropriate-sized endotracheal tube. B) Give chest compressions at a rate of 80 times per minute. C) Administer epinephrine intravenously. D) Suction the mouth and then the nose.

D) Suction the mouth and then the nose.

Which of the following would alert the nurse to suspect that a preterm newborn is in pain? A) Bradycardia B) Oxygen saturation level of 94% C) Decreased muscle tone D) Sudden high-pitched cry

D) Sudden high-pitched cry

Which of the following would the nurse emphasize when teaching postmenopausal women about ways to reduce the risk of osteoporosis? A) Swimming daily B) Taking vitamin A C) Following a low-fat diet D) Taking calcium supplements

D) Taking calcium supplements

A woman experiencing postpartum hemorrhage is ordered to receive a uterotonic agent. Which of the following would the nurse least expect to administer? A) Oxytocin B) Methylergonovine C) Carboprost D) Terbutaline

D) Terbutaline

The nurse teaches a primigravida client that lightening occurs about 2 weeks before the onset of labor. The mother will most likely experience which of the following at that time? A) Dysuria B) Dyspnea C) Constipation D) Urinary frequency

D) Urinary frequency

A pregnant client undergoing labor induction is receiving an oxytocin infusion. Which of the following findings would require immediate intervention? A) Fetal heart rate of 150 beats/minute B) Contractions every 2 minutes, lasting 45 seconds C) Uterine resting tone of 14 mm Hg D) Urine output of 20 mL/hour

D) Urine output of 20 mL/hour

A nursing instructor is describing the various childbirth methods. Which of the following would the instructor include as part of the Lamaze method? A) Focus on the pleasurable sensations of childbirth B) Concentration on sensations while turning on to own bodies C) Interruption of the fear-tension-pain cycle D) Use of specific breathing and relaxation techniques

D) Use of specific breathing and relaxation techniques

After teaching a group of nursing students about the process of involution, the instructor determines that additional teaching is needed when the students identify which of the following as being involved? A) Catabolism B) Muscle fiber contraction C) Epithelial regeneration D) Vasodilation

D) Vasodilation

After teaching new parents about the sensory capabilities of their newborn, the nurse determines that the teaching was successful when they identify which sense as being the least mature? A) Hearing B) Touch C) Taste D) Vision

D) Vision

After teaching a group of students about premenstrual syndrome, the instructor determines that additional teaching is needed when the students identify which of the following as a prominent assessment finding? A) Bloating B) Tension C) Dysphoria D) Weight loss

D) Weight loss

The nurse is making a follow-up home visit to a woman who is 12 days postpartum. Which of the following would the nurse expect to find when assessing the client's fundus? A) Cannot be palpated B) 2 cm below the umbilicus C) 6 cm below the umbilicus D) 10 cm below the umbilicus

a. cannot be palpated

A primipara client gave birth vaginally to a healthy newborn girl 48 hours ago. The nurse palpates the client's fundus, expecting it to be at which location? a. Two fingerbreadths above the umbilicus b. One level of the umbilicus c. Two fingerbreadths below the umbilicus d. Four fingerbreadths below the umbilicus

c. Two fingerbreadths below the umbilicus

The nurse is developing a teaching plan for a client who has decided to bottle feed her newborn. Which of the following would the nurse include in the teaching plan to facilitate suppression of lactation? a. Encouraging the woman to manually express milk b. Suggesting that she take frequent warm showers to soothe her breasts c. Telling her to limit the amount of fluids that she drinks d. Instructing her to apply ice packs to both breasts every other hour

d. instructing her to apply ice packs to both breasts every other hour


Ensembles d'études connexes

AP Euro ch. 19 (French Revolution)

View Set

Waste management potential test ??

View Set